You are on page 1of 59

Neuro-Ophthalmology/Orbit

Question 1 of 130
Which of the following statements does not fully describe chiasmal lesions?

Patients with an empty sella syndrome without a mass or active pseudotumor cerebri may develop chiasmal field
defects.

Meningiomas that affect the chiasm are more often seen in males.

A patient who has had pituitary apoplexy may have normal visual fields.

Optic chiasmal and diabetes insipidus are often associated.


Please select an answer
Feedback: Meningiomas involving the visual system are more common in women than in men, and they may grow during
pregnancy. Trauma to the optic chiasm is often seen in association with both basilar skull fractures and diabetes insipidus
due to the effect of the trauma on the hypothalamic-pituitary axis. Although patients with an empty sella syndrome generally
do not have visual loss, herniation of the chiasm into the empty sella may occasionally cause a chiasmal field defect. In the
pre-MRI era, pituitary apoplexy consisted of the clinical symptom complex of headache and visual loss with or without
hypothalamic-pituitary axis. We now know, however, from MRI that subclinical apoplexy may occur with visual preservation
where the apoplexy (hemorrhage and/or infarction) is an incidental finding.

Question 2 of 130
Which of the following exogenous agents/drugs is not a typical cause of toxic-nutritional optic neuropathy?

Chloramphenicol

Methanol

Hydroxychloroquine sulfate (Plaquenil)

Ethambutol
Please select an answer
Feedback: Toxic-nutritional optic neuropathy is a symmetric process whose clinical profile consists of central or cecocentral
visual field defects, matopsia, and often focal temporal pallor with loss of the nerve fiber layer in the papillomacular bundle.
Because it is a symmetric process, significant RAPD will not occur. This neuropathy may be caused by agents such as
methanol, ethambutol, rifampin, chloramphenicol, and deficiencies in vitamins such as B12, folate, and thiamine.
Hydroxychloroquine sulfate (Plaquenil) causes a maculopathy that may give symmetric central scotomas and cause color
vision loss but does not cause an optic neuropathy.
Question 3 of 130
Which of the following accurately describes optic nerve sheath fenestration and lumboperitoneal shunting in pseudotumor
cerebri?

Optic nerve sheath fenestration should always be performed before lumboperitoneal shunting in pseudotumor cerebri.

Following optic nerve sheath fenestration, low-pressure headaches are never a problem.

Optic nerve sheath fenestration reliably lowers pressure in pseudotumor cerebri.

Lumboperitoneal shunting should always be performed before optic nerve sheath fenestration in pseudotumor cerebri.
Please select an answer
Feedback: With pseudotumor cerebri, optic nerve sheath fenestration will relieve papilledema in the operated eye and, in a
significant percentage of patients, in the contralateral eye. However, it may not reliably reduce intracranial pressure,
headaches, or diplopia. Thus, the degree to which these various symptoms are active and bothersome to the patient will
influence which procedure should be performed. Optic nerve sheath fenestration does not lower intracranial pressure
enough to cause pressure headaches, but these can occur with lumboperitoneal shunting.

Question 4 of 130
Which of the following is optic nerve sheath fenestration not a useful therapeutic intervention for?

Pseudotumor cerebri

Acute anterior ischemic optic neuropathy

Traumatic optic nerve sheath

Papilledema with visual loss from cryptococcal meningitis


Please select an answer
Feedback: Nerve sheath fenestration remains a very useful procedure for visual loss associated with optic nerve head
swelling or compression in all the listed choices except anterior ischemic optic neuropathy. The Ischemic Optic Neuropathy
Decompression Trial demonstrated no benefit with this procedure for anterior ischemic optic neuropathy. With pseudotumor
cerebri, optic nerve sheath fenestration is a procedure of choice from a visual standpoint and is used frequently along with or
in addition to lumboperitoneal shunting. Traumatic optic neuropathy with nerve sheath hematoma, especially in cases where
progressive visual loss is demonstrated, may benefit from optic nerve sheath. With cryptococcal meningitis, it is important to
determine whether the visual loss is due to swelling from increased intracranial pressure (where the procedure is likely to be
beneficial) or due to visual loss secondary to direct cryptococcal invasion of the optic nerve vasculature with secondary
ischemic neuropathy and infarction (where the patients do not benefit from nerve sheath fenestration).
Question 5 of 130
Where is the pathologic problem in Duane's syndrome primarily located?

Third nerve nucleus

Medial longitudinal fasciculus

Cavernous sinus

Sixth nerve nucleus


Please select an answer
Feedback: In the rare cases where tissue has been obtained, the problem appears to arise from an agenesis of the sixth
nerve nucleus, with misdirected innervation from the third nerve.

Question 6 of 130
A 3-month-old child is admitted to the hospital because of drooling and difficulty swallowing. You are asked to see the
patient in consultation, because the child is noted to have complete ophthalmoplegia, ptosis, and pupils that are 9 OU and
unreactive to light or near. What is the most likely diagnosis?

Infantile myasthenia gravis

Infantile botulism

Atropine toxicity

Miller Fisher variant of Guillain-Barré syndrome


Please select an answer
Feedback: This child presents with bulbar weakness (difficulty swallowing with drooling), ptosis, and ophthalmoparesis, with
fixed dilated pupils. Atropine toxicity might give fixed dilated pupils, but it is also often accompanied by tachycardia, flushed
dry skin, fever, and agitation. The affected child would be dry, not drooling, and there would be no ptosis, ophthalmoparesis,
or bulbar weakness. Myasthenia gravis could cause the ptosis, ophthalmoparesis, and bulbar weakness, but pupillary
abnormalities are not seen clinically in this disorder. In the Miller Fisher variant of Guillain-Barré syndrome, which typically
follows a viral illness, eye movement disorders and bulbar weakness may be prominent features, but the pupils and levator
may be unaffected or only mildly involved. This patient has a rather typical presentation for infantile botulism, with the
relatively rapid onset of ophthalmoparesis, ptosis, bulbar weakness, and fixed dilated pupils. This is strongly associated with
the ingestion of honey in children under 1 year of age. A history of honey ingestion should be sought.
Question 7 of 130
Which of the following is not typical of palinopsia?

It is more common with right-sided disease.

It may give the appearance of images persisting after they are gone.

It may give the appearance of multiplicity of objects.

It is characteristic of Meyer's loop lesions.


Please select an answer
Feedback: Palinopsia is an abnormal visual perseveration typically seen with right occipital or parietal (not temporal, ie,
Meyer's loop) lobe disease. Patients perceive an object after it has actually gone from sight. Furthermore, some patients
report seeing multiple images, especially when an object is moving.

Question 7 of 130
Which of the following is not typical of palinopsia?

It is more common with right-sided disease.

It may give the appearance of images persisting after they are gone.

It may give the appearance of multiplicity of objects.

It is characteristic of Meyer's loop lesions.


Please select an answer
Feedback: Palinopsia is an abnormal visual perseveration typically seen with right occipital or parietal (not temporal, ie,
Meyer's loop) lobe disease. Patients perceive an object after it has actually gone from sight. Furthermore, some patients
report seeing multiple images, especially when an object is moving.

Question 8 of 130
A 40-year-old taxi driver complains of loss of vision in his right eye following a whiplash injury. On examination, vision is NLP
OD and 20/15 OS. External and slit-lamp examination are normal. Extraocular movements are full. Pupils are equally
reactive without relative afferent defect, and the examination is normal. What is the most appropriate test for this patient?

MRI

A fluorescein angiogram

Stereo acuity testing

A lumbar puncture
Please select an answer
Feedback: This patient has nonorganic visual loss. Profound subjective monocular visual impairment in the presence of
normally reacting pupils bilaterally and no relative afferent pupillary defect, with normal vision in the contralateral eye, all
suggest that the right eye is normal. Thus, there is no reason to do MRI, a fluorescein angiogram, or a spinal tap. The stereo
acuity testing, along with fogging, OKN testing, and color vision testing with red-green glasses (green in front of the "good"
eye) will all potentially give information about how well the patient actually sees in the reportedly affected eye. Neutral
density filters are useful for quantifying relative afferent pupillary defects; however, as this patient has no such defect, the
filters would not add anything diagnostically.

Question 9 of 130
A 63-year-old right-handed man has an isolated infarct of the entire left parietal lobe. Which of the following would you least
expect to find on examination?

Impaired optokinetic nystagmus with the tape going to the left

Acalculia

Right-left confusion

A right homonymous hemianopsia, worse in the superior hemifield than in the inferior hemifield
Please select an answer
Feedback: Parietal lesions typically cause visual field defects that are worse in the inferior quadrants, as opposed to
temporal lobe lesions, which cause field defects that are worse above (eg, "pie-in-the-sky" scotomas). Gerstmann's
syndrome (ie, acalculia), right-left confusion, finger agnosia (inability to identify one's own fingers), and agraphia, occur with
lesions involving the parietal-occipital junction in the dominant hemisphere. With a left parietal lobe lesion, optokinetic
nystagmus is impaired, because pursuit to the left is impaired.

Question 10 of 130
Which of the following is least consistent with toxic-nutritional optic neuropathy?

A 1.2 log unit relative afferent pupillary defect (RAPD)

Temporal pallor of the optic discs

A low serum B12 and thiamine

Bilateral cecocentral scotomas


Please select an answer
Feedback: Toxic-nutritional optic neuropathy is a symmetric process whose clinical profile consists of central or cecocentral
visual field defects, matopsia, and often focal temporal pallor with loss of the nerve fiber layer in the papillomacular bundle.
Because it is a symmetric process, significant RAPD will not occur. This neuropathy may be caused by agents such as
methanol, ethambutol, rifampin, chloramphenicol, and deficiencies in vitamins such as B12, folate, and thiamine.
Hydroxychloroquine sulfate (Plaquenil) causes a maculopathy that may give symmetric central scotomas and cause color
vision loss but does not cause an optic neuropathy.
Question 11 of 130
A 7-year-old child is seen with unilateral ptosis, mydriasis, and underaction of adduction and vertical movement. Every 2
minutes, the eye adducts, with the lid elevating and the pupil constricting. The mother notes this has been present since
birth. There are no other medical or neurologic problems. What would the best next step be?

Observation

Medical evaluation to rule out occult malignancy

An edrophonium (Tensilon) test

MRI of the brain


Please select an answer
Feedback: Cyclic oculomotor palsy is usually present from birth and has a classic repetitive pattern, generally lasting about
45 seconds, where a cycle of third nerve actions (lid elevation, adduction, pupillary constriction) is superimposed on what
appears to be a third nerve palsy. Cycloplegic migraine, although generally appearing in childhood, occurs with relatively
infrequent, episodic attacks, as with migraine in general. During an attack, a third nerve palsy may occur, which may persist;
however, when it resolves, functions generally return to normal. Periodic alternating nystagmus demonstrates a jerk
nystagmus that beats in one direction, slows, and then reverses direction. There is no evidence of cranial nerve palsy with
this condition. Primary aberrant regeneration is the term used for aberrant regeneration without a history of prior third nerve
palsy. This is generally seen with cavernous sinus lesions such as memingioma or intracavernous carotid aneurysms. It
does not demonstrate the intrinsic rhythmic action seen in our patient. Cyclic oculomotor palsy is not associated with
underlying neurologic problems and thus, especially in this otherwise healthy 7-year-old with a static process since birth, no
workup such as scan or evaluation to rule out occult malignancy is indicated. Observation is sufficient. None of the signs are
at all suggestive of myasthenia gravis, so a Tensilon test is not needed.

Question 12 of 130
A 40-year-old taxi driver complains of loss of vision in his right eye following a whiplash injury. On examination, vision is NLP
OD and 20/15 OS. External and slit-lamp examination are normal. Extraocular movements are full. Pupils are equally
reactive without relative afferent defect, and the examination is normal. Which of the following clinical tests would not be
helpful for the evaluation of the patient?

Color vision testing with red-green glasses

Monocular optokinetic nystagmus (OKN) testing

Testing with neutral density filters

Fogging
Please select an answer
Feedback: This patient has nonorganic visual loss. Profound subjective monocular visual impairment in the presence of
normally reacting pupils bilaterally and no relative afferent pupillary defect, with normal vision in the contralateral eye, all
suggest that the right eye is normal. Thus, there is no reason to do MRI, a fluorescein angiogram, or a spinal tap. The stereo
acuity testing, along with fogging, OKN testing, and color vision testing with red-green glasses (green in front of the "good"
eye) will all potentially give information about how well the patient actually sees in the reportedly affected eye. Neutral
density filters are useful for quantifying relative afferent pupillary defects; however, as this patient has no such defect, the
filters would not add anything diagnostically.
Question 13 of 130
A 34-year-old man presents with acute onset of right facial pain, right ptosis, and diplopia. He also complains of a "funny
feeling" over the right side of his face. On examination, visual acuity is 20/15 OU with full color plates and normal visual
fields. Motility examination shows marked underaction of abduction and upgaze with slow saccades, and absent downgaze
and adduction of the right eye. No torsional movement is seen. The left eye movement is full. Exophthalmometry
measurements are 18 mm on the right and 16 mm on the left. Assuming radiologic, serologic, and pharmacologic tests are
negative, what is the most likely diagnosis?

Tolosa-Hunt syndrome

Dysthyroid orbitopathy

Chronic progressive external ophthalmoplegia

Occult tumor
Please select an answer
Feedback: Tolosa-Hunt syndrome is the eponym for an idiopathic cavernous sinus inflammatory syndrome. It is probably a
posterior variation of orbital pseudotumor. Tolosa-Hunt is often described as "exquisitely responsive," although it is the pain
that responds quickly. The motility may take weeks or months to improve. An occult tumor might be considered, but a lesion
large enough to involve cranial nerves III-VI completely would be visualized on an imaging study and would not present so
acutely. It would be unlikely for a carcinomatous meningitis to involve so many contiguous unilateral cranial nerves. A
remote effect of malignancy may cause brain stem findings, not discrete cranial nerve palsies. Dysthyroid orbitopathy may
cause some discomfort, but would not present with pain. It would not have the associated right facial dysesthesia, especially
involving the CN V2 and CN V3 distribution. Chronic progressive external ophthalmoplegia is a painless, bilaterally
symmetric motility problem without sensory involvement.

Question 14 of 130
A 32-year-old woman complains of intermittent diplopia, usually lasting about half an hour when she first wakes up in the
morning, with a few months' duration. She also notes some irritation and pressure around her eyes. Acuity, color plates, and
visual fields are all normal. Externally, the right upper eyelid is at the upper limbus. The left covers the upper 2 mm of the
cornea. There is slight injection OU with some chemosis on the left. Versions are normal, but cover testing demonstrates a
small left hypertropia (or right hypotropia) on up gaze. Corneal sensation is normal. Several months later, the left eyelid of
this patient becomes slightly ptotic, and she develops a 12-PD exotropia at distance, with a moderate underaction of
adduction of the right eye. Horizontal forced testing is normal. What is the most likely diagnosis?

Internuclear ophthalmoplegia

Myasthenia gravis

Chronic progressive external ophthalmoplegia

Worsening dysthyroid ophthalmopathy


Please select an answer
Feedback: The clinical appearance of ptosis with exotropia and the underaction of the medial rectus would be most
suspicious for myasthenia gravis. Thus, a Tensilon test would be indicated. Thyroid disease and myasthenia may occur
concomitantly. In this case, however, the underaction of the medial rectus is the major abnormality of motility; this would be
unusual in dysthyroid disease because the lateral rectus is the least likely of the extraocular muscles to be involved.
Underactions in dysthyroid orbitopathy are due to a restrictive process in the antagonist muscle, and this patient's forced test
was negative. As noted previously, eyelid retraction is common; therefore, the presence of ptosis should alert the physician
to the possibility of a second process. A Tensilon test should be performed measuring any change in deviation, motility, and
eyelid position. The thyroid status does not reflect the orbital disease status, so repeat thyroid function tests would not be
indicated. Chronic progressive external ophthalmoplegia (CPEO) is a bilateral, generally symmetric process without diplopia.
The patient presented clearly has an asymmetric process. Muscle biopsy would not be the most appropriate test at this point
given the preponderance of evidence for concomitant myasthenia and the clinical appearance being so inconsistent with
CPEO. Internuclear ophthalrnoplegia would not necessarily be associated with a unilateral ptosis and would show a slow
adducting saccade (not mentioned in the findings just noted), as well as an abducting nystagmus of the contralateral eye.
Thus, an MRI is not indicated.

Question 15 of 130
A 34-year-old man presents with acute onset of right facial pain, right ptosis, and diplopia. He also complains of a "funny
feeling" over the right side of his face. On examination, visual acuity is 20/15 OU with full color plates and normal visual
fields. Motility examination shows marked underaction of abduction and upgaze with slow saccades, and absent downgaze
and adduction of the right eye. No torsional movement is seen. The left eye movement is full. Exophthalmometry
measurements are 18 mm on the right and 16 mm on the left. What would the most appropriate test be?

MRI of the head, with gadolinium

An edrophonium (Tensilon) test

CT of orbits, with contrast

Cerebral angiogram
Please select an answer
Feedback: The “funny feeling" over the right side of the face (sensory trigeminal nerves CN V2 to CN V3) puts the lesion
posterior to the orbit. The involvement of cranial nerves III, IV, V, and VI points to the cavernous sinus. Thus, MRI of the
head with attention to the cavernous sinus is indicated, rather than CT of the orbits. Posterior communication artery
aneurysms may cause pain and a third nerve palsy, but wouldn't involve the fourth, fifth, and sixth cranial nerves; thus, a
cerebral angiogram would not be immediately indicated. An intracavernous carotid artery aneurysm might have these
findings, but would probably be seen on MRI, which should be done first. A Tensilon test to rule out myasthenia gravis is
clearly not indicated, as myasthenia gravis does not cause pain or lead to sensory complaints.

Question 16 of 130

A 7-year-old child is seen with unilateral ptosis, mydriasis, and underaction of adduction and vertical movement. Every 2
minutes, the eye adducts, with the lid elevating and the pupil constricting. The mother notes this has been present since
birth. There are no other medical or neurologic problems. What are these signs representative of?

Cyclic oculomotor palsy

Periodic alternating nystagmus

Ophthalmoplegic migraine

Primary aberrant regeneration


Please select an answer
Feedback: Cyclic oculomotor palsy is usually present from birth and has a classic repetitive pattern, generally lasting about
45 seconds, where a cycle of third nerve actions (lid elevation, adduction, pupillary constriction) is superimposed on what
appears to be a third nerve palsy. Cycloplegic migraine, although generally appearing in childhood, occurs with relatively
infrequent, episodic attacks, as with migraine in general. During an attack, a third nerve palsy may occur, which may persist;
however, when it resolves, functions generally return to normal. Periodic alternating nystagmus demonstrates a jerk
nystagmus that beats in one direction, slows, and then reverses direction. There is no evidence of cranial nerve palsy with
this condition. Primary aberrant regeneration is the term used for aberrant regeneration without a history of prior third nerve
palsy. This is generally seen with cavernous sinus lesions such as memingioma or intracavernous carotid aneurysms. It
does not demonstrate the intrinsic rhythmic action seen in our patient.
Question 17 of 130
A 59-year-old man complains of decreased vision immediately upon emerging from anesthesia following cardiac bypass
surgery. On examination, the patient reports bare light perception vision OU. External examination, extraocular movements,
pupillary reaction, and examination are all normal. Which of the following is the most likely cause of the visual loss?

Pituitary apoplexy

Functional or nonorganic visual loss

Bilateral anterior ischemic optic neuropathy

Bilateral occipital infarcts


Please select an answer
Feedback: The two most common causes of visual loss following bypass surgery are occipital infarction and ischemic optic
neuropathy, with the former being much more common. With bilateral anterior ischemic optic neuropathy (AION) and
pituitary apoplexy, both of which affect the anterior visual pathway, such severe visual loss would cause poorly reactive
pupils. In addition, with AION, on ophthalmoscopic examination the optic nerve heads would appear swollen. With pituitary
apoplexy, there may also be complaints of headache and limitation of extraocular movements due to involvement of the
cranial nerves in the cavernous sinus. To consider a functional or nonorganic visual loss, you would need to demonstrate
that the vision is actually better than reported. For example, you would need to demonstrate optokinetic nystagmus
response or see the patient perform purposeful, visually directed tasks. In this patient, occipital infarction (bilaterality should
be assumed because of the decreased visual acuity), should be presumed.

Question 18 of 130
A 59-year-old man complains of decreased vision immediately upon emerging from anesthesia following cardiac bypass
surgery. On examination, the patient reports bare light perception vision OU. External examination, extraocular movements,
pupillary reaction, and examination are all normal. Which of the following is the diagnostic procedure of choice?

Carotid duplex study

Erythrocyte sedimentation rate

MRI or CT scan

Fluorescein angiogram
Please select an answer
Feedback: MRI is the procedure of choice with occipital infarction. However, patients may not be medically stable enough to
undergo this, in which case CT should be performed. Fluorescein angiography would not likely be helpful with normal optic
nerves and fundi. Visual loss in patients with occipital infarction is usually due to hypoperfusion of the occipital lobes during
the bypass procedure, which involves the vertebrobasilar system; thus a carotid duplex study is not indicated. Patients
poststernotomy and post-bypass surgery have high erythrocyte sedimentation rates. Therefore, this test would be too
unreliable and potentially misleading regarding the possibility of temporal arteritis as the cause, and is not indicated.
Question 19 of 130
A 68-year-old man complains of severe, brief, episodic, electric shock-like pain on the left side of his face, mostly over the
cheek. It may be precipitated by chewing or shaving. He is otherwise asymptomatic. Ophthalmologic and neurologic
evaluation are normal. What is the most likely diagnosis?

Temporal arteritis

Postherpetic neuralgia

Cavernous sinus aneurysm

Trigeminal neuralgia
Please select an answer
Feedback: Based on the triggering factors and the nature of the pain, this is a classic presentation of trigeminal neuralgia. In
a younger age group, consideration should be given to a structural lesion or even demyelinating disease as a cause of
trigeminal neuralgia. Temporal arteritis must be considered as well in this patient and a determination made as to whether
the inciting factors could perhaps represent skin tenderness or jaw claudication. Temporal arteritis, however, causes
headaches, and sometimes systemic symptoms (fever, anorexia, polymyalgia). Also, the elicited pain would not be so brief
or resemble electric shock. Although cavernous sinus aneurysms may also cause severe pain, the pain is more persistent
and less likely to have inciting factors. In addition, there are no other signs suggesting a cavernous sinus problem.
Postherpetic neuralgia is excluded, as no history of zoster is given and no skin changes are noted. The pain of postherpetic
neuralgia is also more continuous and deeper in nature and may be associated with numbness.

Question 20 of 130
A 32-year-old woman complains of intermittent diplopia, usually lasting about half an hour when she first wakes up in the
morning, with a few months' duration. She also notes some irritation and pressure around her eyes. Acuity, color plates, and
visual fields are all normal. Externally, the right upper eyelid is at the upper limbus. The left covers the upper 2 mm of the
cornea. There is slight injection OU with some chemosis on the left. Versions are normal, but cover testing demonstrates a
small left hypertropia (or right hypotropia) on up gaze. Corneal sensation is normal. Which of the following is the most
appropriate next test?

Acetylcholine receptor antibody test

Thyroid function test

FTA-ABS

MRI of the brain


Please select an answer
Feedback: This patient has the classic symptoms and soft tissue signs of dysthyroid orbitopathy. In the early stages, it may
present with intermittent diplopia, generally worse in the morning, secondary to orbital congestion. The eyelid findings
suggest subtle retraction OD because the OS position is normal, as described, not ptotic. Myasthenia gravis, although
frequently presenting with intermittent diplopia, would not have all these subjective complaints (pressure, irritation) or the lid
retraction. However, the possibility of coexisting myasthenia gravis and Graves' disease should at least be considered.
Pituitary tumors without visual field loss wouldn't be expected to cause intermittent diplopia, especially with this morning
predominance, nor would it cause injection and chemosis. A cavernous sinus syndrome may show injection and chemosis
(although it may also show a perfectly white eye), but it wouldn't show the lid retraction or the diurnal variation. A cavernous
sinus syndrome might also have more motility disturbance, as well as CN V1 sensory complaints. The majority of patients
with dysthyroid orbitopathy will be hyperthyroid on presentation, although they may be hypothyroid or euthyroid. Thyroid
function tests are indicated to evaluate the patient's overall status, in addition to trying to confirm the diagnosis. A suggested
screening panel would include sensitive TSH, total T, and thyroid-stimulating immunoglobulins. Acetylcholine receptor
antibodies might be indicated if myasthenia gravis were the most likely choice. If an imaging study were to be undertaken, it
should be an orbital study (CT or MRI) rather than a routine brain study, as listed. As this does not appear to be a true
cranial neuropathy or a discrete inflammatory lesion, FTA-ABS would not be indicated.
Question 21 of 130
A 1-year-old child is referred for evaluation of "jiggling eyes." The mother notes that this condition has been present since
about 1month of age. On examination, the child fixes and follows well with either eye. The anterior segment is normal.
Retinoscopy is +2.50+0.50 x 90 degrees OD; +2.75+0.25 x 75 degrees OS. Dilated fundus examination is normal.
Extraocular movements are full. You note binocular horizontal pendular nystagmus that diminishes on right gaze and when
the patient fixates at near. General medical and neurologic evaluation is reportedly normal. What is the most likely finding on
examination?

Normal vertical optokinetic nystagmus response

An abnormal fundus on subsequent evaluations

Light-near dissociation on pupillary testing

Convergence retraction nystagmus on upgaze


Please select an answer
Feedback: This child has motor congenital nystagmus demonstrated by a null point and damping at near. The condition is
not always appreciated right at birth. On testing, horizontal optokinetic nystagmus (OKN) responses may or may not be
elicited or may show inversion; that is, instead of the recovery saccade (the “jerk” phase of nystagmus) being in the
expected opposite direction to the drum or OKN tape rotation, it is in the same direction. Vertical saccades, however, are
generally preserved, which yields a somewhat oblique nystagmus when the OKN response is generated, as vertically
induced saccades are superimposed on the horizontal baseline nystagmus. Convergence retraction nystagmus is seen with
the dorsal midbrain syndrome. One would not expect to see a primary horizontal nystagmus with a null point in this entity.
Normal motility, especially vertical, also argues against a dorsal midbrain syndrome. No other ocular abnormalities were
noted. Light-near dissociation may be seen as part of the dorsal midbrain syndrome, but this is also seen when there is
damage to the afferent visual system. There is no evidence of either optic nerve or retinal disease, and the child does fix and
follow well. The normal anterior segment and retina excludes albinism. A patient with Leber 's congenital amaurosis may
have a normal fundus on early evaluation although abnormality evolves over time. In addition, one does not get a null point
and damping at near with Leber's.

Question 22 of 130
An obese 26-year-old woman with a 6-month history of pseudotumor cerebri who is on acetazolamide (Diamox) 500 mg po
qid, notes progressive blurring of vision OS over the past few weeks. She denies headaches or double vision. Examination
shows chronic papilledema. Visual fields are stable on the right and show progressive constriction on the left. Which of the
following interventions is most desirable at this time?

Weight reduction

Optic nerve sheath fenestration OS

Repeat lumbar punctures

Lumboperitoneal shunt
Please select an answer
Feedback: Although treatment of choice remains controversial, most ophthalmologists would agree that optic nerve sheath
fenestration should be recommended in this setting. The patient has progressive visual problems only in one eye.
Lumboperitoneal shunting could be performed, although given the potential risks and problems associated with this
procedure, it is not the first choice here. Because spinal fluid is completely replenished over 8 hours, repeat spinal taps are
not indicated. Weight loss should be encouraged, but in the face of progressive visual loss, a more definitive intervention,
optic nerve sheath fenestration, is indicated.
Question 23 of 130
A 32-year-old woman complains of intermittent diplopia, usually lasting about half an hour when she first wakes up in the
morning, with a few months' duration. She also notes some irritation and pressure around her eyes. Acuity, color plates, and
visual fields are all normal. Externally, the right upper eyelid is at the upper limbus. The left covers the upper 2 mm of the
cornea. There is slight injection OU with some chemosis on the left. Versions are normal, but cover testing demonstrates a
small left hypertropia (or right hypotropia) on up gaze. Corneal sensation is normal. What is the most likely diagnosis at this
time?

Dysthyroid orbitopathy

Cavernous sinus syndrome

Myasthenia gravis

Pituitary tumor
Please select an answer
Feedback: This patient has the classic symptoms and soft tissue signs of dysthyroid orbitopathy. In the early stages, it may
present with intermittent diplopia, generally worse in the morning, secondary to orbital congestion. The eyelid findings
suggest subtle retraction OD because the OS position is normal, as described, not ptotic. Myasthenia gravis, although
frequently presenting with intermittent diplopia, would not have all these subjective complaints (pressure, irritation) or the lid
retraction. However, the possibility of coexisting myasthenia gravis and Graves' disease should at least be considered.
Pituitary tumors without visual field loss wouldn't be expected to cause intermittent diplopia, especially with this morning
predominance, nor would it cause injection and chemosis.

Question 24 of 130
A patient's granddaughter incidentally noted that his pupils were unequal. The patient denies ptosis or diplopia. In bright
light, his pupils are 1 mm OD and 3 mm OS. In dim light, his pupils are 1 mm OD and 4 mm OS. The right pupil is unreactive
to light or near stimulus. Which of the following best explains the clinical findings?

History of recent chiropractic manipulation

History of prior, treated unilateral glaucoma

History of blindness OD secondary to optic neuropathy

History of syphilis as a young man


Please select an answer
Feedback: Treatment of unilateral glaucoma with pilocarpine will produce a small pupil that is poorly reactive to light or
near. Chiropractic manipulation associated with anisocoria is typically due to a Horner syndrome from a carotid artery
dissection and would produce a small pupil that is reactive to light but has poor dilation and a greater degree of anisocoria in
the dark. Syphilis can produce a tonic pupil or the Argyll-Robertson pupil but both produce light near dissociation of the pupil
response. A history of blindness secondary to an optic neuropathy would produce a poor light reaction but the pupil would
retain the near reaction if there was afferent disease only.
Question 25 of 130
A 64-year-old hypertensive patient complains of difficulty reading. He states, "Things are blurry, I sometimes lose my place
when reading, and I still see things after I've looked away." Examination demonstrates acuity of 20/20 OU with normal
motility, pupillary, and fundus evaluation. What would the most appropriate initial evaluation be?

Psychiatric consultation

Visual field test

Erythrocyte sedimentation rate

Fluorescein angiogram
Please select an answer
Feedback: Any patient with persistent visual complaints despite a normal routine eye examination should have visual field
testing. The symptom of losing his place while reading should suggest a left hemianopia. The complaint of still seeing things
after looking away represents palinopsia, a visual release phenomenon seen with hemianopias, most commonly right
parietal lesions. The likelihood of temporal arteritis causing visual loss is unlikely with a normal fundus. Similarly, fluorescein
angiography in the setting of a normal funduscopic examination would not be the first test recommended. A psychiatric
consultation is not indicated unless a nonorganic visual loss is demonstrated. The other field defects would not cause the
constellation of visual symptoms described.

Question 26 of 130
A 20-year-old nurse in an ophthalmology clinic complains of right-sided brow ache. Examination of both eyes is normal
except for the presence of a 1.5 mm, nonreactive right pupil. The pupil does not dilate in darkness. The left pupil is 4 mm
and briskly reactive. She has no ptosis and normal ocular motility. The anterior chamber has no cell and flare. What is the
most likely etiology?

Exposure to a cholinergic (pilocarpine-like) substance in the right eye

Exposure to an anticholinergic (atropine-like) substance in the right eye

Horner's syndrome in the right eye

Uveitis
Please select an answer
Feedback: A cholinergic exposure will prduce a pupil that is miotic and nonreactive to light or near. Atropine or any
anticholinergic substance will produce a mydriatic pupil, not a miotic pupil. The Horner syndrome can produce a small pupil,
but this pupil will still respond to light and near stimuli. Uveitis can produce a small and poorly reactive pupil, but the
absenece of cell and flare make this less likely, although this is a potentially correct answer
Question 27 of 130
Which of the following features is most consistent with pharmacologic pilocarpine use in a patient with anisocoria?

Small pupil is unreactive to light or near

Small pupil is unreactive to light but reactive to near

Hyperopic shift

Abnormal pupillary sphincter appearance


Please select an answer
Feedback: Pharmacologic iris blockade with pilocarpine, which is a parasympathomimetic, would constrict the pupil and
produce poor or no reaction to light or near stimuli. A small pupil that is unreactive to light but reactive to near is a sign of the
Adie’s tonic pupil (i.e., the "little old Adie’s pupil"). The pupil sphincter would remain round from parasympathomimetic
stimulation equally to the pupil, and an irregular pupil is a sign of the Adie’s tonic pupil or a structural defect in the iris itself.
A myopic rather than hyperopic shift occurs with pilocarpine use.

Question 28 of 130
A 64-year-old hypertensive patient complains of difficulty reading. He states, "Things are blurry, I sometimes lose my place
when reading, and I still see things after I've looked away." Examination demonstrates acuity of 20/20 OU with normal
motility, pupillary, and fundus evaluation. What might the visual field examination be expected to show?

Bitemporal hemianopia

Left homonymous hemianopia

An altitudinal field defect in the dominant eye

Binasal hemianopia
Please select an answer
Feedback: Any patient with persistent visual complaints despite a normal routine eye examination should have visual field
testing. The symptom of losing his place while reading should suggest a left hemianopia. The complaint of still seeing things
after looking away represents palinopsia, a visual release phenomenon seen with hemianopias, most commonly right
parietal lesions. The likelihood of temporal arteritis causing visual loss is unlikely with a normal fundus. Similarly, fluorescein
angiography in the setting of a normal funduscopic examination would not be the first test recommended. A psychiatric
consultation is not indicated unless a nonorganic visual loss is demonstrated. The other field defects would not cause the
constellation of visual symptoms described.
Question 29 of 130
A 20-year-old nurse in an ophthalmology clinic complains of right-sided brow ache. Examination of both eyes is normal,
except for the presence of a 1.5 mm, nonreactive right pupil to light or near stimuli. The pupil does not dilate in darkness.
The left pupil is 4 mm and briskly reactive. The patient has no ptosis and normal ocular motility. The anterior chamber has
no cell and flare. What is the most likely diagnosis?

Horner's syndrome

Argyll Robertson pupil

Adie's pupil

Instillation of miotic agent


Please select an answer
Feedback: A pupil that is poorly reactive or nonreactive to light or near stimuli and does not dilate suggests iris damage or
pharmacologic miosis with a topical parasympathomimetic. A Horner syndrome produces a small pupil that remains reactive
to light and near. The Adie’s tonic pupil will react poorly or not at all to light, but will have a better and tonic near response.
The Argyll Robertson pupil is a bilateral, small, irregular pupil with a poor light reaction, but a better near reaction (light near
dissociation of the pupils).

Question 30 of 130
A 15-year-old girl experiences blurry vision OS and a frontal headache. Her acuities are 20/20 OD and 20/60 without
correction OS. With -2.00 D refraction she sees 20/20 OS. In the light, pupils measure 6 mm OD and 1 mm OS, and in the
dark they measure 7 mm OD and 1 mm OS. The right pupil is briskly reactive; the left does not react to light or near stimuli.
Her examination is otherwise unremarkable. What type of drop did she use?

Parasympathomimetic

Prostaglandin inhibitors

Sympathomimetic

Alpha adrenergic agonist


Please select an answer
Feedback: A sympathomimetic drop would dilate the pupil, not constrict the pupil. It would not produce a myopic shift, and
the pupil would remain reactive to light and near stimuli. A prostaglandin inhibitor would have no effect on the pupil, and an
alpha adrenergic agent would have the same effect as a sympathomimetic. Only a parasympathomimetic drop would
constrict the pupil and induce the myopic shift.
Question 31 of 130
A 70-year-old woman reports double vision and "trouble with my eyes." This apparently developed suddenly, a few months
ago. Visual acuity, color vision, and threshold visual field evaluation are normal. Extraocular movements show no adduction
OD and no abduction or adduction OS; motility is otherwise normal. Pupillary evaluation is normal. There is no ptosis.
External examination is unremarkable. Which of the following tests is most indicated?

MRI of the parasellar area

Aan edrophonium (Tensilon) test

MRI of the head/pons

CT of the orbit
Please select an answer
Feedback: This motility pattern describes a one-and-a-half syndrome, which is a gaze palsy (the "one") with a contralateral
internuclear ophthalmoplegia (the “half”). It represents a lesion in the pons, most commonly with a demyelinating cause in
younger patients and an ischemic cause in older patients, such as this one. MRI to determine the cause is indicated in this
patient with no pertinent history. Myasthenia gravis can mimic any eye movement disorder, although it would be unusual to
have such a complete one-and-a-half syndrome without any other extraocular muscle or levator involvement. Myasthenia
would also not be expected to arise so suddenly. Thus, a Tensilon test is not indicated. Dysthyroid orbitopathy may cause
underaction of abduction, although having this pattern without other extraocular muscle involvement would be very unusual.
The sudden onset and normal external examination also argue against this diagnosis; as a result, orbital CT scanning is not
indicated. Regarding bilateral cavernous sinus syndrome, one might view this patient's problem as a sixth nerve palsy with a
partial, bilateral third nerve involvement. However, one does not get complete involvement of one muscle innervated by the
third nerve (medial rectus in this case) without any involvement of either lid, pupil, or other extraocular muscles on a
peripheral basis, only centrally; thus, MRI with attention to the parasellar area would not be indicated.

Question 32 of 130
A 24-year-old woman complains of episodes of blurred vision with flashing lights that progress over approximately 30
minutes. These are followed by a throbbing headache and resolution of her visual symptoms. A complete ophthalmologic
evaluation, including visual fields, is normal. The most likely diagnosis was classic migraine. On a follow-up visit, the patient
notes eight additional episodes. In all cases, she reports, the blur and scintillations occurred only in the left visual field, with
headaches on the right side. She denies any other accompanying neurologic symptoms. Assuming all testing proved
negative, daily prophylactic therapy would not include which of the following?

A beta blocker

Dihydroergotamine (DHE)

Amitriptyline

A calcium channel blocker


Please select an answer
Feedback: All of the medicines listed, except dihydroergotamine, may be used in migraine prophylaxis. DHE may be used
for acute migraine attacks, but, because of the toxic side effects of ergot, it is essential not to overprescribe this drug. The
most common side effects are nausea and vomiting. Longer-term use can produce peripheral vascular constriction, causing
such problems as pain and loss of pulses in the extremities, angina, and other ischemic changes.
Question 33 of 130
A 62-year-old woman presents complaining that "everything is jumping." Visual acuity, color vision, and visual fields are
normal. Motility evaluation shows full movement. You note a coarse, downward beating of both eyes, worse on downgaze,
especially laterally. What would this sign most likely represent?

Periodic alternating nystagmus

Superior oblique myokymia

Downbeat nystagmus

Myasthenia gravis
Please select an answer
Feedback: Downbeat nystagmus is a bilateral, downward beating nystagmus, classically worse on down gaze and lateral
gaze downward. Superior oblique myokymia is unilateral, usually with very fine oscillations, and worsens into the field of the
superior oblique muscle (down and in). Periodic alterating nystagmus is a horizontal jerk nystagmus that beats in one
direction, slows, stops, and then reverses direction. Myasthenia gravis rarely presents as nystagmus.

Question 34 of 130
Which of the following statements relates to a patient who has acute optic neuritis in the right eye?

With the information given, the results of the relative afferent pupillary defect test cannot be predicted.

It would be rare for the patient to have a new visual field defect on automated perimetry in the left eye.

There must be a relative afferent pupillary defect present in the right eye.

There cannot be a better pupillary response in the right eye than in the left.
Please select an answer
Feedback: Optic neuritis frequently is a sign of multiple sclerosis. Many patients with optic neuritis have a prior history of
contralateral optic neuritis. If this patient had had a prior optic neuritis in the left eye, a relative afferent pupillary defect
(RAPD) might not be present when acute optic neuritis occurred in the right eye. Indeed, if optic nerve function were worse
in the left eye than in the right at the time of examination, the RAPD might be in the left eye and the pupil might respond
better in the right eye. In the case described, therefore, with no information about the contralateral eye, the results of the
RAPD testing cannot be predicted. At the time of acute optic neuritis, contralateral asymptomatic visual field defects are
commonly seen.
Question 35 of 130
A 22-year-old woman exhibits a dilated right pupil that you clinically suspect is an Adie's tonic pupil. The pupils are 8 mm
OD, 5 mm in bright light, and 8 mm OS, 2 mm in bright light. The motility is normal, and there is no ptosis. While preparing
the appropriate pharmacologic solutions, you take a more detailed history. Which of the following historical points would be
of the least significance in a patient who demonstrates a large pupil that does not constrict?

The patient has a history of motion sickness and has just returned from a cruise.

The patient says that the right eye with the larger pupil has always had a lighter colored iris than the left eye.

The patient is aware of an inability to read with the right eye only.

The patient is an lCU nurse who runs cardiac resuscitation codes.


Please select an answer
Feedback: The question is, What in the history is consistent with having an ipsilateral dilated pupil? That the patient is an
ICU nurse is relevant because during cardiac resuscitation, ampules of atropine are used in haste, and a resuscitation team
member might become subject to atropinization by inadvertent hand-eye contact. Inability to read due to loss of ipsilateral
accommodation is a symptom of Adie's tonic pupil. The most effective pharmacologic treatment of motion sickness is
transdermal scopolamine, which may cause ipsilateral pupillary dilation by direct inoculation from the fingers. A lighter iris in
the affected eye is insignificant for two reasons: pupils in Horner's syndrome do respond to light; furthermore, even if this
patient had a congenital Horner's syndrome, the lighter iris would be ipsilateral to the smaller pupil.

Question 36 of 130
Which of the following statements does not accurately describe optokinetic response testing?

In a patient with a complete right adduction deficit, ability to cause the right eye to fully adduct in response to an
optokinetic drum indicates that the ophthalmoparesis is not infranuclear in origin.

Absence of response with the tape moving to the patient's left may indicate a left frontal lobe lesion.

Presence of optokinetic nystagmus response to an optokinetic drum at a test distance of 3 feet indicates a visual acuity
of 20/60 or better.

Absence of response with the tape moving to the patient's right may indicate a right parietal lesion.
Please select an answer
Feedback: The optokinetic response may be thought of as a pursuit movement in the direction of the movement of the drum
or tape, and a refixational saccadic movement in the direction away from the movement of the drum or tape. In the case of a
drum moving to the patient's right, the following pursuit movement toward the right is mediated by the right parietal lobe. The
refixational leftward saccade is mediated by the right frontal lobe. Thus, optokinetic response testing may be thought of as
testing the saccade and pursuit initiation in the frontal and parietal lobes on the side toward which the tape or drum is
moving. As with caloric or oculocephalic maneuvers, optokinetic response testing may be used to demonstrate that the final
common pathway from the nucleus to nerve to extraocular muscle is intact, or that the ocular rotation defect is supranuclear
in origin. An optokinetic drum may also be used to estimate visual acuity (as in cases of functional visual loss), but the
minimum acuity requisite to obtain a response to a drum at 3 feet is only counting fingers at 3 to 5 feet.
Question 37 of 130
An emergent consultation is requested on a comatose man with significant periocular trauma to the right eye. A CT scan of
the head is essentially normal. The patient's pupils are 8 mm OD, 4 mm OS; the right pupil does not react to direct light. The
right eye is exodeviated and has complete ptosis, and oculocephalic maneuver indicates that the right eye does not cross
the midline in the field of action of the right medial rectus muscle. There is no enophthalmos or proptosis. Intraocular
pressures are 19 mm Hg OD and 12 mm Hg OS. Both fundi are unremarkable. For the purpose of guiding emergent
treatment, which of the following is the most appropriate test to perform?

Forced duction of the right eye

A CT scan of the orbits

Examination of the relative magnitude of the pupillary response of the left eye, when the left and then the right eye is
illuminated

A pattern visual evoked response


Please select an answer
Feedback: In the trauma patient, determination of the status of the optic nerves is of paramount importance. In this
comatose patient with an apparent traumatic right pupil-involving oculomotor palsy, the direct response of the right pupil
cannot be used to evaluate right optic nerve function. However, a constrictive response of the left pupil when the light is
swung from the right eye to the left eye, and a dilation of the right pupil when the light is swung back to the right eye, would
indicate that the right optic nerve is carrying less afferent information than the left optic nerve. This is the only immediately
available bedside method of assessing the right optic nerve function.

Question 38 of 130
A 24-year-old woman complains of episodes of blurred vision with flashing lights that progress over approximately 30
minutes. These are followed by a throbbing headache and resolution of her visual symptoms. A complete ophthalmologic
evaluation, including visual fields, is normal. What is the most likely diagnosis?

Classic migraine

Superior oblique myokymia

Transient ischemic attack

Cluster headache
Please select an answer
Feedback: These are the symptoms of a classic migraine, or migraine with aura. The visual symptoms are cortical in origin,
so this is not a retinal or ophthalmic migraine (both of the latter are truly monocular events). Cluster headaches are episodes
of severe pain in and around one eye and side of the head, often associated with tearing and nasal congestion. They tend to
occur daily over a limited period of time (the "cluster") and then resolve. Transient ischemic attacks are generally more
abrupt at onset, and are less likely to cause a positive phenomenon, such as flashing lights; rather, they will present with a
defect, such as a hemianopia. In addition, they are not likely to be followed by headache or to follow such a stereotypic
course. Superior oblique myokymia may be perceived as a "shimmering" of vision, but it occurs monocularly, doesn't follow
a stereotypic time course, and is not associated with headaches.
Question 39 of 130

A 17-year-old boy receives an ophthalmic examination before beginning driver's education. The results of the examination
are as follows: Uncorrected acuity: 20/20 OU; Pupils: 5 mm OU, round and reactive; mild RAPD OD; Motility: Normal; Slit
lamp: Normal; Intraocular pressures: 16 mm Hg OD, 18mm Hg OS; Visual fields: Inferior altitudinal defect OD, normal OS.
Which of the following points in the history or examination, if elicited, would not aid in arriving at a diagnosis?

A cilioretinal artery is present OD only.

He has optic nerve hypoplasia OD, and his mother has insulindependent (juvenile-onset) diabetes mellitus.

A superior hemiretinal detachment is present OD.

The patient had an episode of marked acute visual loss OD with pain on ocular rotation 2 years before, with partial
recovery.
Please select an answer
Feedback: Offspring of mothers with insulin-dependent (juvenile-onset) diabetes mellitus may have optic nerve head
hypoplasia with an associated congenital altitudinal scotoma. Optic neuritis and a hemiretinal detachment could also cause
an altitudinal scotoma. The finding of a cilioretinal artery has no bearing on the presence of the altitudinal field defect.

Question 40 of 130
A 24-year-old woman complains of episodes of blurred vision with flashing lights that progress over approximately 30
minutes. These are followed by a throbbing headache and resolution of her visual symptoms. A complete ophthalmologic
evaluation, including visual fields, is normal. The most likely diagnosis was classic migraine. On a follow-up visit, the patient
notes eight additional episodes. In all cases, she reports, the blur and scintillations occurred only in the left visual field, with
headaches on the right side. She denies any other accompanying neurologic symptoms. Which of the following is the most
appropriate course of action?

Observation only

Echocardiogram

Carotid ultrasound

MRI
Please select an answer
Feedback: With multiple recurrent episodes of the same system, all on the same side, a fixed, structural lesion, especially
an arteriovenous malformation mimicking migraine, must be ruled out. MRI is the procedure of choice. With so many
episodes all in the same distribution, observation alone would not be appropriate. The episodic nature might suggest an
ischemic cause, but the purely visual symptoms without other neurologic symptoms would suggest a posterior,
vertebrobasilar system distribution (not the carotids) as the most likely localization; thus, a carotid ultrasound is not the first
choice of test. A cardiac cause as a possible source of emboli could be considered, but with multiple identical episodes
suggesting only one site of involvement (and no symptoms suggesting other sites of embolization) an echocardiogram would
not be considered appropriate at this time.
Question 41 of 130
A mass causing a relative afferent pupillary defect and a homonymous hemianopia would most likely be found in which one
of the following?

Optic nerve

Optic chiasm

Optic tract

Occipital lobe
Please select an answer
Feedback: Occipital lobe lesions are not associated with pupillary abnormalities. Optic nerve (unilateral visual field defect)
and chiamsal problems (temporal visual field defects) do not result in a homonymous hemianopia. Optic tract lesions cause
a combination of a homonymous hemianopia, and a relative afferent pupillary defect.

Question 42 of 130
Which one of the following is a contraindication to magnetic resonance imaging?

Orbital wood foreign body

Iodine allergy

Acute hemorrhage

Cochlear implant
Please select an answer
Feedback: Cochlear implantation is a contraindication to MRI. Iodine allergy may be a contraindication to CT scan dye
injection. Wood foreign bodies and acute hemorrhage are not contraindications to either type of imaging study.

Question 43 of 130
A 25-year-old man is found to have a normal visual field in the right eye and a complete temporal hemianopic defect in the
left eye. The visual field is repeated under binocular conditions, and the left hemianopic visual field defect persists. These
findings are most consistent with which one of the following?

Optic tract syndrome

Retinopathy

Chiasmal syndrome

Nonphysiologic visual loss


Please select an answer
Feedback: An optic tract syndrome would cause a homonymous hemianopia. A nasal retinopathy on the left may cause a
temporal defect, however, the binocular visual field would be normal. The normal (nasal) visual field of the right eye would
still be able to compensate for any potential temporal visual field loss of the left eye.
Question 43 of 130
A 25-year-old man is found to have a normal visual field in the right eye and a complete temporal hemianopic defect in the
left eye. The visual field is repeated under binocular conditions, and the left hemianopic visual field defect persists. These
findings are most consistent with which one of the following?

Optic tract syndrome

Retinopathy

Chiasmal syndrome

Nonphysiologic visual loss


Please select an answer
Feedback: An optic tract syndrome would cause a homonymous hemianopia. A nasal retinopathy on the left may cause a
temporal defect, however, the binocular visual field would be normal. The normal (nasal) visual field of the right eye would
still be able to compensate for any potential temporal visual field loss of the left eye.

Question 44 of 130
Which of the following findings would most likely suggest a supranuclear gaze palsy in a 72-year-old man who cannot look
down?

Normal doll’s head maneuver

Pupillary light-near dissociation

Bilateral abduction deficits

Downbeating nystagmus
Please select an answer
Feedback: Downbeating nystamus may occur irrespective to a gaze palsy. Abduction deficits do not reveal whether the
cause is nuclear or suparnuclear in origin. Light-near dissociation suggests a problem in the midbrain. A normal doll’s head
maneuver suggests the nuclear and peripheral pathways for eye movement are functional.

Question 45 of 130
A 22-year-old woman with acute visual loss in the right eye and pain with eye movement from optic neuritis due to multiple
sclerosis is most likely to have which one of the following clinical findings?

Optic disc edema with macular star

Uveitis with vitritis

Relative afferent pupillary defect

Anisocoria greater in light


Please select an answer
Feedback: The description would be most consistent with an inflammatory optic neuropathy, when asymmetric or unilateral,
is associated with a relative afferent pupillary defect. An optic neuropathy does not cause anisocoria. Optic disc edema with
a macular star (neuroretinitis) and uveitis are not typically seen in multiple sclerosis.
Question 46 of 130
What is the most likely diagnosis in a 48-year-old man who awakens after cardiac bypass surgery with decreased vision in
the right eye, a right relative afferent pupillary defect, and normal funduscopy?

Posterior ischemic optic neuropathy

Optic neuritis

Cilioretinal artery occlusion

Occipital stroke
Please select an answer
Feedback: The findings suggest a right optic neuropathy. The presence of a relative afferent pupillary defect excludes an
occipital lobe lesion. Normal funduscopy excludes chorioretinopathy. The only choice for an optic neuropathy listed is
posterior ischemic optic neuropathy, which indicates that the optic disc is normal acutely and develops pallor weeks after the
ischemia.

Question 47 of 130
Which one of the following disorders of facial activity is most commonly bilateral?

Essential blepharospasm

Eyelid myokymia

Hemifacial spasm

Idiopathic (Bell’s) facial palsy


Please select an answer
Feedback: Hemifacial spasm is characteristically unilateral and persists during sleep. Idiopathic facial palsy and eyelid
myokymia are most commonly unilateral. Essential blepharospasm is bilateral and abolished by sleep.

Question 48 of 130
Eye pain with ipsilateral ptosis would be most likely to be seen in a patient with which one of the following?

Thyroid eye disease

Optic nerve sheath meningioma

Carotid dissection

Orbital cavernous hemangioma


Please select an answer
Feedback: Thyroid eye disease is the most common cause of eyelid retraction. Orbital cavernous hemangioma, optic nerve
sheath meningioma, and other orbital disorders most commonly cause proptosis. Carotid dissection may cause ipsilateral
Horner syndrome, which is characterized by painful ptosis.
Question 49 of 130
Which one of the following would be expected in a patient with ocular ischemic syndrome?

Retinal venous attenuation

Horner syndrome

Nyctalopia

Carotid stenosis
Please select an answer
Feedback: Ocular ischemic syndrome, or venous stasis retinopathy, is characterized by venous dilation, typically in the
absence of tortusoity, retinal hemorrages, and decreased vision after exposure to light, likely secondary to prolonged
recovery of the retinal photoreceptors due to ischemia. Internal carotid stenosis (typically 90% or more) is the most common
cause of ocular ischemic syndrome, but Horner syndrome would not be an expected finding.

Question 49 of 130
Which one of the following would be expected in a patient with ocular ischemic syndrome?

Retinal venous attenuation

Horner syndrome

Nyctalopia

Carotid stenosis
Please select an answer
Feedback: Ocular ischemic syndrome, or venous stasis retinopathy, is characterized by venous dilation, typically in the
absence of tortusoity, retinal hemorrages, and decreased vision after exposure to light, likely secondary to prolonged
recovery of the retinal photoreceptors due to ischemia. Internal carotid stenosis (typically 90% or more) is the most common
cause of ocular ischemic syndrome, but Horner syndrome would not be an expected finding.

Question 50 of 130
A 42-year-old woman has anisocoria that is greater in light. The right pupil is sluggishly reactive to light and she has normal
ocular motility, alignment, and normal eyelid position. What would be the best next step?

10% cocaine eye drops

MRI/MRA of the brain

Slit-lamp biomicroscopy

Carotid ultrasonography
Please select an answer
Feedback: The findings suggest a problem with the pupillary sphincter (greater anisocoria in light). Normal lid position,
alignment, and eyelid position make third nerve palsy unlikely. Cocaine is useful in the diagnosis of Horner syndrome. Slit-
lamp biomicroscopy would be the most helpful to distinguish abnormalities in the iris that might suggest a tonic pupil or
trauma as the cause of anisocoria.

Question 51 of 130
A 335-pound 25-year-old woman comes in on a Monday for evaluation of bilateral synchronous visual obscurations lasting
15 seconds. These have been occurring for 3 weeks. She has no other medical history and is on no medications. Her
examination is normal except for enlarged blind spots on visual field testing and marked bilateral papilledema. What is the
most reasonable next step in her management?

Start her on acetazolamide 500 mg q6h, and refer her to a neurologist on Tuesday.

Obtain an MRI that day; if it is negative, begin her on acetazolamide 500 mg q6h and see her in 3 days to determine
whether the papilledema has improved.

Start her on acetazolamide 500 mg q6h, and refer her to a neurologist on Thursday.

Obtain an MRI on her that day; if it is negative, arrange for a spinal tap shortly thereafter.
Please select an answer
Feedback: Even in the patient whose presentation is classic for pseudotumor cerebri, two critical steps must be taken
before the diagnosis can be made. First, it is imperative to rule out the presence of an intracranial mass, which can be a
neurosurgical emergency and can certainly occur in the obese patient. MRI is the imaging method of choice; it also serves to
establish that the pseudotumor cerebri is not secondary to dural sinus thrombosis. If there is no mass, the second requisite
for establishing the diagnosis of pseudotumor cerebri is that the spinal tap show raised intracranial pressure with normal
chemical and cellular composition of the spinal fluid (eg, no evidence of inflammation or infection). Only after these steps are
taken is it appropriate to begin therapy for pseudotumor cerebri.

Question 52 of 130
Which of the following best describes Leber's hereditary optic neuropathy?

Females may develop Leber's hereditary optic neuropathy.

The onset in the two eyes is always simultaneous.

It is purely a clinical diagnosis, based on the family inheritance pattern.

It is rare for affected patients to have no family history of visual loss.


Please select an answer
Feedback: Although males are most often affected (80% to 90% of cases in Caucasians, 60% of cases in Japanese),
females are affected more often than was previously reported. Males at genetic risk have a higher chance of developing
visual loss than do females at genetic risk for Leber's hereditary optic neuropathy (LHON). We now know that LHON is a
phenotypic expression of several different mitochondrial genetic alterations. These mitochondrial genetic sequences can
now be established, allowing for laboratory confirmation in nearly all cases. Worldwide, the 11778 point mutation accounts
for about 50% of all LHON cases. The 3460 point mutation accounts for 8% to 25% of LHON cases. A third point mutation at
locus 14484 may cause 10% of LHON. Many LHON patients have a negative family history. The interval between the
involvement of the first and the second eye is usually days to weeks, but it may extend to months or years in some cases.
Question 53 of 130
A 62-year-old woman complains that "everything is jumping." Visual acuity, color vision, and visual fields are normal. Motility
evaluation shows full movement. You note a coarse, downward beating of both eyes, worse on downgaze, especially
laterally. What is the anatomic site most likely to be involved?

Medial longitudinal fasciculus

Cervicomedullary/cerebellar area

Parasellar/third ventricle

Dorsal midbrain ventricle


Please select an answer
Feedback: Downbeat nystagmus is frequently associated with problems at the cervicocomedullary junction, such as Arnold-
Chiari malformations or cerebellar atrophy. MRI with attention to this area is indicated. Observation probably is not
adequate; one should try to discern whether a structural basis exists for the problem. MRA is indicated for vascular lesions.
Because this patient's problem generally does not arise from an aneurysm or vascular malformation, MRA is not indicated in
this case. A Tensilon test is also not indicated, as myasthenia would not present as nystagmus. Dorsal midbrain lesions are
associated with convergence retraction nystagmus, as well as weakness of up gaze, possibly down gaze, and light-near
dissociation. There may also be abduction problems. Parasellar/third ventricle lesions are associated with see-saw
nystagmus. The medial longitudinal fasciculus is involved in internuclear ophthalmoplegia, which clinically consists of an
underaction of adduction with nystagmus in the contralateral abducting eye.

Question 54 of 130
A 45-year-old right-handed male executive who is an avid golfer presents with the complaint that for the past year, whenever
he addresses a golf ball, it appears to be shimmering up and down, with perhaps a rotatory component to it. Sometimes the
ball looks double to him in the vertical plane. The patient, a good observer, says that the image moves and is double only
when viewed with the right eye. The results of his examination are normal. Which of the following is the most appropriate
action to take?

Perform an edrophonium (Tensilon) test

Start the patient on carbamazepine (Tegretol)

Obtain a psychiatric consultation

Start the patient on phenytoin (Dilantin)


Please select an answer
Feedback: The patient has given a classic description of superior oblique myokymia. In this disorder, there is an intermittent
rapid oscillopsia, which is largely a cyclorotatory phenomenon, with intorsional movements. These episodes may be
triggered by moving the eye into the field of action of the affected superior oblique muscle. When a right-handed golfer
addresses a golf ball; the eye moves into a depressed and adducted position; which can trigger these attacks.
Carbamazepine (Tegretol) seems to help most patients with this condition, although some report that the beneficial effect is
transitory.
Question 55 of 130
A 3-year-old girl is brought in by her mother for evaluation of a mass that is medial and superior to the left medial canthus
that has been present since birth. The mother is not sure if it has been growing. The examination is normal, except for
nystagmus OU. You have difficulty visualizing the fundus. The child is short for her age; otherwise, she is doing well. Which
of the following would not be an appropriate management at this point?

Needle biopsy of the mass, followed by neuroimaging if not inflammatory

Examination under anesthesia

Neuroimaging

Pediatric endocrinologic consultation


Please select an answer
Feedback: Septo-optic dysplasia (de Morsier's syndrome) consists of a triad of growth failure, optic nerve hypoplasia, and
nystagmus. Neuroimaging serves two purposes. First, it will demonstrate the classic finding of an absent septum pellucidum
(the so-called upside-down Liberty Bell sign). Second, as these dysplasias may be associated with basal encephaloceles
and other midline abnormalities, neuroimaging will be needed to demonstrate that the mass is not an encephalocele before
any biopsy is performed. In a child with unexplained nystagmus where the fundus cannot be well visualized in the
examination room, examination under anesthesia is a reasonable part of the ophthalmologist's evaluation. Because patients
with septo-optic dysplasia may have hypothalamic dysfunction (including deficiencies of growth hormone,
adrenocorticotropic hormone, and antidiuretic hormone), endocrinologic evaluation is part of their assessment.

Question 56 of 130
A 70-year-old woman reports double vision and "trouble with my eyes." This apparently developed suddenly, a few months
ago. Visual acuity, color vision, and threshold visual field evaluation are normal. Extraocular movements show no adduction
OD and no abduction or adduction OS; motility is otherwise normal. Pupillary evaluation is normal. There is no ptosis.
External examination is unremarkable. What is the most likely diagnosis?

One-and-a-half syndrome

Dysthyroid orbitopathy

Bilateral cavernous sinus syndrome

Myasthenia gravis
Please select an answer
Feedback: This motility pattern describes a one-and-a-half syndrome, which is a gaze palsy (the "one") with a contralateral
internuclear ophthalmoplegia (the “half”). It represents a lesion in the pons, most commonly with a demyelinating cause in
younger patients and an ischemic cause in older patients, such as this one. MRI to determine the cause is indicated in this
patient with no pertinent history. Myasthenia gravis can mimic any eye movement disorder, although it would be unusual to
have such a complete one-and-a-half syndrome without any other extraocular muscle or levator involvement. Myasthenia
would also not be expected to arise so suddenly. Thus, a Tensilon test is not indicated. Dysthyroid orbitopathy may cause
underaction of abduction, although having this pattern without other extraocular muscle involvement would be very unusual.
The sudden onset and normal external examination also argue against this diagnosis; as a result, orbital CT scanning is not
indicated. Regarding bilateral cavernous sinus syndrome, one might view this patient's problem as a sixth nerve palsy with a
partial, bilateral third nerve involvement. However, one does not get complete involvement of one muscle innervated by the
third nerve (medial rectus in this case) without any involvement of either lid, pupil, or other extraocular muscles on a
peripheral basis, only centrally; thus, MRI with attention to the parasellar area would not be indicated.
Question 57 of 130
A 68-year-old man complains only of recent onset of "trouble reading." There is no past medical history. The results of the
patient's examination are as follows: Best-corrected acuity: distance-20/25 OU, near-20/20 OU; Pupils: 3 mm OU, round and
reactive; no RAPD; lid fissures: 10 mm OU; Motility: Horizontal eye movement normal, volitional down gaze severely and
symmetrically limited, up gaze mildly limited. Oculocephalic maneuver completely overcomes the vertical eye movement
deficiency; Slit lamp: Normal; Intraocular pressures: Normal; Visual fields: Normal; Fundi: Normal. Which of the following
would be most inconsistent with the diagnosis of the patient?

Extensor neck rigidity

Square wave jerks

Dementia

Pigmentary retinopathy
Please select an answer
Feedback: Progressive dementia, square wave jerks (small amplitude saccadic jerky movements away from fixation),
ataxia, and axial rigidity are all typical in the course of progressive supranuclear palsy. Pigmentary retinopathy is a feature of
the Kearns-Sayre syndrome, which is in the differential of a progressive symmetric ophthalmoparesis, but would not have
late onset and would not be likely to respond to the oculocephalic maneuver

Question 58 of 130
A 62-year-old woman presents complaining that "everything is jumping." Visual acuity, color vision, and visual fields are
normal. Motility evaluation shows full movement. You note a coarse, downward beating of both eyes, worse on downgaze,
especially laterally. Which of the following actions is most recommended?

Observation

Magnetic resonance imaging (MRI)

An edrophonium (Tensilon) test

Magnetic resonance angiography (MRA)


Please select an answer
Feedback: Downbeat nystagmus is frequently associated with problems at the cervicocomedullary junction, such as Arnold-
Chiari malformations or cerebellar atrophy. MRI with attention to this area is indicated. Observation probably is not
adequate; one should try to discern whether a structural basis exists for the problem. MRA is indicated for vascular lesions.
Because this patient's problem generally does not arise from an aneurysm or vascular malformation, MRA is not indicated in
this case. A Tensilon test is also not indicated, as myasthenia would not present as nystagmus. Dorsal midbrain lesions are
associated with convergence retraction nystagmus, as well as weakness of up gaze, possibly down gaze, and light-near
dissociation. There may also be abduction problems. Parasellar/third ventricle lesions are associated with see-saw
nystagmus. The medial longitudinal fasciculus is involved in internuclear ophthalmoplegia, which clinically consists of an
underaction of adduction with nystagmus in the contralateral abducting eye.
Question 59 of 130
A patient involved in a motor vehicle accident had loss of consciousness for 10 minutes. She has an abduction deficit of the
right eye, with only about 10% of normal amplitude of abduction beyond the midline remaining. The anterior examination is
normal; exophthalmometry readings are symmetric. Which of the following would be consistent with the presence of a right
sixth-nerve palsy without an entrapped medial rectus?

Ability to induce full abduction OD with oculocephalic maneuver

Restriction of the right eye on forced abduction, no force generated with attempted abduction on forced generation test

No restriction on forced abduction OD, diminished force generated with attempted abduction on forced generation test

Exodeviation on right gaze


Please select an answer
Feedback: In significant head trauma, it is possible to see both a neurogenic sixth-nerve palsy from the closed head injury
and medial rectus entrapment from an orbital fracture. In the setting of a medial orbital fracture with medial rectus
entrapment and without sixth-nerve dysfunction, the forced duction would demonstrate resistance opposite to the field of
action of the restricted muscle. In a nearly complete sixth-nerve paresis, one would not be able to completely overcome the
abduction deficit with oculocephalic maneuver. Also, in an isolated acute right sixth-nerve paresis, there would be an
esodeviation in all fields of gaze. In an acute sixth-nerve paresis without entrapment, there would be no resistance to forced
aductions, but there would be either no force or reduced force imparted to the right globe in right abduction, because the
right lateral rectus is not receiving normal neural input to induce its contraction.

Question 60 of 130
A 67-year-old Asian man consulted you 3 months ago with an isolated right sixth-nerve palsy that had been present for 3
months. His CT scan with contrast of the head, edrophonium (Tensilon) test, glucose tolerance test, sedimentation rate and
serologies for Lyme disease and syphilis were normal. He now presents with a left sixth-nerve palsy to accompany the
persistent right sixth-nerve palsy. The results of the rest of the examination remain normal. There is no proptosis. A spinal
tap performed by his neurologist 1 day earlier was normal. What is the most likely location of the pathologic process?

Genu of right facial nerve around abducens nucleus

Both orbital apices

Clivus

Bilateral abducens nucleus


Please select an answer
Feedback: Both sixth nerves come together to ascend the clivus. The sequential involvement of both abducens nerves
should alert the ophthalmologist to consider a mass lesion in this location. Although a process involving both orbital apices
could theoretically cause bilateral sixth-nerve palsies, proptosis and other signs of orbital involvement would be expected. A
lesion of the genu of the right facial nerve might give an ipsilateral sixth- and seventh-nerve paresis, but not bilateral sixth-
nerve palsies. Bilateral nuclear sixth-nerve palsies would give the patient bilateral gaze paresis.
Question 61 of 130
A 67-year-old Asian man consulted you 3 months ago with an isolated right sixth-nerve palsy that had been present for 3
months. His CT scan with contrast of the head, edrophonium (Tensilon) test, glucose tolerance test, sedimentation rate, and
serologies for Lyme disease and syphilis were normal. He now presents with a left sixth-nerve palsy to accompany the
persistent right sixth-nerve palsy. The results of the rest of the examination remain normal. There is no proptosis. A spinal
tap performed by his neurologist 1 day earlier was normal. Which of the following is the least important in terms of his
management plan?

Perform another spinal tap in a few days

Obtain an otolaryngology consult

Obtain MRI of the base of the brain

Obtain cervical carotid ultrasonography


Please select an answer
Feedback: In an Asian male presenting with bilateral sixth-nerve palsies, nasopharyngeal carcinoma is a concern, as the
ophthalmoparesis may be the presenting sign from tumor extension along the clivus. Otolaryngologic consultation is thus
indicated. Multiple spinal taps may be required to detect carcinomatous meningitis; thus, repeat spinal tap is appropriate. As
the CT scan of the head may not image the base of the brain well, MRI is indicated. Carotid atherosclerotic disease would
not present in this manner.

Question 62 of 130

Which of the following syndromes with neuro-ophthalmic signs is not associated with the endocrinopathy mentioned?

De Morsier's syndrome/growth failure

Chiasmal trauma/diabetes insipidus

Chiasmal sarcoidosis/Cushing's disease

Angiomatosis retinae (von Hippel-Lindau disease)/pheochromocytoma


Please select an answer
Feedback: Due to concomitant trauma to the hypothalamic-pituitary axis, chiasmal trauma is often associated with diabetes
insipidus. Septo-optic dysplasia, or de Morsier's syndrome, consists of agenesis of the septum pellucidum, optic nerve
hypoplasia, and growth failure. Angiomatosis retinae may have several extraocular manifestations, including
hemangioblastoma of the central nervous system; renal, pancreatic, hepatic, or epididymal cysts; and pheochromocytoma.
Sarcoidosis of the chiasm is often accompanied by diabetes insipidus, not Cushing's disease, due to the attendant
hypothalamic infiltration and dysfunction.
Question 63 of 130
A patient presents with complete bilateral ophthalmoplegia that has lasted 1 week. The lid fissures are 3 mm OU. The pupils
are 3 mm OU, round, reactive to light, and without RAPD. The afferent visual system is unaffected, and the patient can read
with either eye without difficulty. The patient had a viral episode with diarrhea, nausea, and vomiting 2 days before the onset
of the ocular symptoms. Despite having one eye patched, she is ataxic and areflexic. The rest of the examination is normal.
Which of the following statements would not describe this occurance?

The patient probably does not have botulism.

The patient should undergo spinal tap.

The patient might have Miller Fisher syndrome.

The patient probably has myasthenia.


Please select an answer
Feedback: This is another classic neuro-ophthalmic triad, that is, ophthalmoplegia, areflexia, and ataxia. This defines the
Miller Fisher variant of Guillain-Barre syndrome. Botulism is an unlikely diagnosis without either pupillary involvement or
blurred vision at near. Myasthenia would not cause areflexia or ataxia. The spinal fluid findings of a marked rise in protein
without cellular infiltrate (pleocytosis) help confirm the clinical diagnosis of Miller Fisher syndrome.

Question 64 of 130

Which of the following findings would be unusual in a patient presenting for an examination before craniotomy for a large
pineal tumor?

Convergence-retraction nystagmus only on attempted saccades downward

Skew deviation

Light-near dissociation of the pupils

Lid retraction
Please select an answer
Feedback: In Parinaud's syndrome (also known as dorsal midbrain syndrome, or sylvian aqueduct syndrome) the pupils
may not react to light, but will react to near fixation. These pupils tend to be midsize or large, and of greater size than those
seen with syphilis. Lid retraction (Collier's sign) is typical, and skew deviation may be part of the findings. In this disorder,
there is a selective involvement of up gaze, and, early on, saccades may be affected more than pursuit. Attempts at upward
saccades may induce convergence-retraction nystagmus. Isolated convergence-retraction nystagmus on attempted down
gaze is not a feature of this syndrome.
Question 65 of 130
A 7-year-old boy is referred by a pediatrician for evaluation of his visual status. The child is said to be slow in school and is
being placed in special education. The pediatrician wishes to ensure that a refractive error is not contributing to the poor
performance. The child has had three seizures, and a raised papular rash in a butterfly distribution has been noted on his
cheeks. Some patchy hypopigmented areas are also seen on the trunk. The child is otherwise well. What is the most likely
diagnosis?

Juvenile-onset systemic lupus erythematosus with cerebritis

Tuberous sclerosis (Bourneville's disease)

Neurofibromatosis type I (Von Recklinghausen's disease)

Encephalotrigeminal angiomatosis (Sturge-Weber syndrome)


Please select an answer
Feedback: All of the entities listed are capable of causing fundus abnormalities and seizures with cutaneous findings.
However, the triad of mental retardation, seizures, and the rash described in a butterfly distribution (adenoma sebaceum,
actually angiofibromas) is most consistent with the diagnosis of tuberous sclerosis, or Bourneville's disease. This autosomal
dominant disorder also has a typical CT scan finding of calcifications from intracerebral astrocytic hamartomas. Periungual
fibromas may be seen, as well as cutaneous hypopigmented "ash-leaf" spots. Sturge-Weber syndrome has a different
cutaneous manifestation, the typical nevus flammeus. Lupus erythematosus can cause seizures from cerebritis, but other
clinical signs would probably be seen; in addition, the rash is different and would not include "ash-leaf" spots.
Neurofibromatosis would not have a raised rash, and the rash would not be in this distribution.
Question 66 of 130
A patient presents with optic neuropathy and uveitis with mutton-fat keratic precipitates. The patient is noted to have bilateral
large, nodular lacrimal glands and a white raised lesion on the involved optic nerve head. Which of the following test results
would be most likely to exclude the diagnosis of sarcoidosis in this patient?

A positive PPD (purified protein derivative), with the rest of the anergy panel negative (normal cutaneous response was
present to mumps)

A normal chest x-ray

A normal conjunctival biopsy

A normal ACE (angiotensin converting enzyme)


Please select an answer
Feedback: A patient may have ocular involvement of sarcoidosis without having an abnormal chest x-ray. Similarly, the
ACE is typically not elevated unless there is pulmonary disease. Positive conjunctival biopsies are useful in making the
diagnosis, as they provide histologic confirmation of the diagnosis, which is preferred when one needs to institute systemic
therapy for sarcoidosis. However, many patients with systemic sarcoidosis will have normal conjunctival biopsies,
particularly if no lesion is clinically visible. Patients with tuberculosis may, on occasion, have an elevated ACE and certainly
would have an abnormal chest x-ray. They may also have optic nerve involvement and uveitis. Patients with sarcoidosis
usually show lack of response (anergy) on the anergy panel, whereas tuberculosis patients will typically have positive
response to cutaneous testing with PPD. A patient with this presentation and a positive PPD must be considered to have
tuberculosis and not sarcoidosis.

Question 67 of 130
A 37-year-old woman presents with ptosis and ophthalmoplegia. She first noted ptosis at age 12, and noted trouble moving
her eyes 3 years later. It has been progressive over the years; she never had any diplopia, and she believes that her vision,
once her eyelids are raised, is normal. There is no family history. She denies other medical problems, and she has not seen
a physician for 10 years. Which of the following would not be an appropriate action to take at the initial visit?

Send off mitochondrial genetic analysis

Perform edrophonium (Tensilon) test

Schedule cardiac evaluation

Schedule muscle biopsy


Please select an answer
Feedback: Besides ophthalmoplegia and retinopathy, patients with Kearns-Sayre syndrome should also have at least one
of the following other three manifestations for the clinical diagnosis to be made: cardiac conduction abnormalities (ie, heart
block), elevated cerebrospinal fluid protein, or cerebellar dysfunction. The clinical diagnosis is supported by muscle biopsy
demonstrating "ragged red fibers”. Mitochondrial genetic analysis shows gene deletions in many cases, confirming the
diagnosis. Spinal fluid analysis is also useful, because elevated cerebrospinal fluid protein is a criterion for diagnosis.
Because cardiac conduction abnormalities (including heart block, which may cause sudden death) may be a feature of
Kearns-Sayre syndrome, insertion of a pacemaker is often part of the management of these cases. Thus, edrophonium
(which may cause bradycardia) should not be given to a patient in whom Kearns-Sayre syndrome is in the differential
diagnosis until the patient has had a cardiac evaluation.
Question 68 of 130
Two patients present with bilateral internuclear ophthalmoplegia. One is 20-years-old, and the other is 65. The younger
patient has no other medical history, and the 65-year-old has always been well, but recently diagnosed with hypertension.
Their examinations are otherwise normal. Which of the following statements regarding either patient would not be
appropriate?

The older patient probably has a vasculopathic process.

The older patient should be treated with 1 g of methylprednisolone sodium succinate (Solumedrol) daily for 3 days.

The most likely problem in the younger patient is delnyelinating disease.

The problem in both patients localizes to the pons.


Please select an answer
Feedback: Bilateral internuclear ophthalmoplegia localizes to the pons. In the young, a common cause is multiple sclerosis.
In the older age group, where atherosclerotic lesions are common, the etiology is often vasculopathic. For a 65-year-old to
have this presentation as an isolated first sign of demyelination would be distinctly unusual, and further evidence for
demyelination would need to be found before embarking on a trial of corticosteroids.

Question 69 of 130
Which of the following historical points, or physical findings, would most call for MRI in a patient who is having an eleventh
bout of a classic migraine syndrome?

The visual phenomena are homonymous

The patient is 17 years old

Hemianopia is always on the same side

The patient is a male


Please select an answer
Feedback: Migraine is not uncommon at age 17, and although migraine headaches are more common in females, they are
not unusual in males. In classic migraine, the visual phenomenon (scintillating scotoma) is typically homonymous. Although
these episodes may be rather stereotypical, and they most often may be on one particular side of the head, always having
the symptoms on one side raises the question of a fixed lesion causing the migrainous phenomenon, such as an
arteriovenous malformation.
Question 70 of 130
A 64-year-old woman reports progressive onset of ptosis and diplopia over an 8-month period. On examination, the left eye
is normal, but the right eye reveals the following abnormalities: 4 mm of ptosis, limitation of eye movement in all directions, 4
mm pupil that does not react to light or dilate in darkness, and loss of corneal sensation. What is the most likely diagnosis?

Graves' ophthalmopathy

Myasthenia gravis

Intracavernous meningioma

Aneurysm of the posterior communicating artery


Please select an answer
Feedback: The patient's findings can best be explained by involvement of the third, sixth, fifth, and sympathetic nerves.
Fourth-nerve involvement cannot be determined from the description. A mid-dilated pupil that does not react to light or dilate
in darkness usually indicates impairment of both the parasympathetic and the sympathetic innervation of the pupil. This
finding, as well as the other findings, can best be explained by a lesion in the cavernous sinus. Meningioma and internal
carotid artery aneurysm are the most common causes, particularly with a slowly progressive history. Myasthenia gravis and
Graves' ophthalmopathy would not produce the pupillary or sensory findings noted in this patient. An aneurysm of the
posterior communicating artery could produce a third-nerve palsy, but would not be expected to produce the other findings.
A midbrain lesion typically produces bilateral ptosis, if any, and could not explain the sensory findings.

Question 71 of 130
Which one of the following conditions would most likely be found in a patient with optic disc edema, macular exudate, and
vitritis?

Optic neuritis

Hypertensive retinopathy

Nonarteritic anterior ischemic optic neuropathy

Papilledema
Please select an answer
Feedback: Inflammatory optic neuropathy (optic neuritis) would be expected to be associated with other evidence of ocular
inflammation in some cases. None of the other conditions would be expected to be associated with inflammation.
Question 72 of 130
Transient visual loss in one eye associated with eye pain lasting one hour is most consistent with which of the following?

Cardiac embolic disease

Vertebrobasilar arterial insufficiency

Papilledema

Uveitis-glaucoma-hyphema syndrome
Please select an answer
Feedback: Uveitis-glaucoma-hypema typically causes symptoms which last longer than transient visual loss from
thromboembolic disease. None of the other conditions is typically associated with eye pain when the transient visual loss
occurs.

Question 73 of 130
Which of the following findings would be expected to be present one year after a stroke involving the right optic tract?

Right relatve afferent pupillary defect

Right homonymous hemianopia

Right optic disc pallor

Right dyschromatopsia
Please select an answer
Feedback: Optic tract lesions cause bilateral optic disc pallor in association with a contralateral homonymous hemianopia
and a contralateral relative afferent pupillary defect.

Question 74 of 130
Which of the following would be most suggestive of a compressive lesion causing a right sixth cranial nerve palsy?

Progressive diplopia over a 6-month period

Esotropia worse in left gaze

Right periorbital pain

Intermittent diplopia
Please select an answer
Feedback: Compressive lesions tend to cause progressive visual symptoms with the continued growth of the mass.
Question 75 of 130
An abduction deficit caused by a sixth cranial nerve palsy can be best distinguished from that due to thyroid eye disease by
which one of the following?

Forced duction testing

A double maddox rod

An exophthalmometer

Alternate cover testing


Please select an answer
Feedback: Restrictive processes can be differentiated from paralytic processes using forced duction testing. None of the
other techniques listed would help differeniate between these two.

Question 76 of 130
Which one of the following findings would be most likely in a patient with unilateral proptosis and eyelid retraction?

Retinal arterial vasculitis

Cystoid macular edema

Pars planitis

Choroidal folds
Please select an answer
Feedback: Thyroid eye disease would not be expected to be associated with inflammation in the eye or cystoid macular
edema. Thyroid eye disease is one of the most common causes of choroidal folds, which are often horizontal with respect to
the macula.

Question 77 of 130
Papilledema would be an expected cause of visual loss in a patient with which of the following findings?

Meningeal enhancement from intracranial hypotension

Bilateral optic nerve sheath enhancement

Pituitary apoplexy

Hydrocephalus from aqueductal stenosis


Please select an answer
Feedback: Papilledema, by defintion, is optic disc swelling from elevated intracranial pressure. Only hydrocephalus would
be expected to cause papilledema.
Question 78 of 10
Difficulty reading in a patient with a Chiari malformation would be most likely from which of the following findings?

Bitemporal hemianopia

Downbeating nystagmus

Skew deviation

Fourth cranial nerve palsy


Please select an answer
Feedback: Difficulty reading in a patient with a Chiari malformation would be most likely from downbeating nystagmus.

Question 79 of 130

Which of the following findings would be most suggestive of an optic neuropathy in a 50-year-old woman with decreased
vision in the right eye?

Decreased visual acuity

Decreased color vision

Central scotoma

Relative afferent pupillary defect


Please select an answer
Feedback: Of the choices listed, a relative afferent pupillary defect would be the most suggestive of an optic neuropathy.
Cataract and/or macular degeneration could cause the other problems listed.

Question 80 of 130
Which of the following findings would be most suggestive of visual loss from optic disc drusen?

Dyschromatopsia

Central scotoma

Choroidal folds

Inferonasal visual field defect


Please select an answer
Feedback: Optic disc drusen is associated with central visual loss or dyschromatopsia in most patients. Inferonasal defects
are the most common type of visual field loss from optic disc druen, which do not cause mass effect resulting in choroidal
folds.
Question 81 of 130

A 60-year-old man has intermittent binocular horizontal diplopia. Physical examination reveals impaired adduction of the left
eye with an abducting nystagmus of the right eye. For a single lesion to explain these findings where is its location?

Right medial longitudinal fasciculus

Left third-nerve fascicle

Left medial longitudinal fasciculus

Right sixth-nerve fascicle


Please select an answer
Feedback: Internuclear ophthalmoplegia is characterized by an adduction deficit and contralateral abducting
nystagmus. Damage to the medial longitudinal fasciculus, which extends from the pons to the midbrain, is responsible for
causing an internuclear ophthalmoplegia. Thus, this constellation of findings would be a LEFT, not a RIGHT, medial
longitudinal fasciculus. A lesion of the 6th or 3rd fasciculus would not be transitory, bilateral, or associated with nystagmus.

Question 82 of 130
A 53-year-old woman has difficulty keeping her eyes open. A visit to her primary care provider reveals nothing. She tells you
that it is easier to keep her eyes open if she hums or sings, touches her temples or her forehead between her eyes, or
chews on her glasses. What condition does this most likely represent?

Levator dehiscence

Myasthenia gravis

Benign essential blepharospasm

Dry eye
Please select an answer
Feedback: Benign essential blepharospasm is typically bilateral and often is aggravated by periods of stress. Myasthenia
gravis may cause bilateral ptosis, which would be expected to worsen with increased activity of the facial muscles, such as
during prolonged upgaze, and worsen throughout the day.

Question 83 of 130
Temporal arteritis is diagnosed on biopsy when which of the following findings is present?

Lymphocyes and plasma cells

Intimal hyperplasia

Calcific plaque with arterial occlusion

Granulomatosis inflammation with or without giant cells


Please select an answer
Feedback: The hallmark histopathologic finding in temporal (giant) cell arteritis is granulomatous inflammation. Giant cells
need not be present. There may also be disruption of the internal elastic lamina and thicking of the intima, but these may
also be seen in conditions other than temporal arteritis. Intimal hyperplasia or lymphocytes and plasma cells are not specific
for temporal arteritis. Calcific plaque with arterial occlusion is characteristic of embolic disease from atherosclerosis.

Question 84 of 130
A 28-year-old man complains of painless loss of vision in his right eye. On exam he is 20/20 OD with 1 diopter of hyperopic
shift, and 20/20 OS. Color saturation is equal between both eyes, and there is no relative afferent pupillary defect. After
static automated perimetry, the patient's central sensitivities are mildly reduced on the right, and he has a normal visual field
on the left. Photo stress recovery time is 2 minutes on the right and 20 seconds on the left. What is the most likely
diagnosis?

Central serous retinopathy

Vitreous hemorrhage

Retrobulbar optic neuritis

Amblyopia
Please select an answer
Feedback: Maculopathies are associated with a prolonged (> 50 seconds) photostress recovery, which is thought to occur
because of prolonged recovery of dysfunctional retinal photoreceptors. Absence of relative afferent pupillary defect supports
that it is unlikely that this is due to an optic neuropathy. Hyperopia may develop as the retina is elevated, and is not typical of
an isolated optic neuropathy or amblyopia.

Question 85 of 130
A 30-year-old man with chronic sinus disease has endoscopic sinus surgery. Postoperatively, he develops a retrobulbar
hemorrhage. Physical examination reveals no damage to the globe. What is the most urgent indication for lateral
canthotomy/cantholysis?

Intraocular pressure > 50 mm Hg with a relative afferent pupillary defect

Presence of subconjunctival hemorrhage

Limited ocular motility with diplopia

Onset of proptosis
Please select an answer
Feedback:

The most pressing indication for surgical intervention in a patient with orbital hemorrhage is visual loss from optic
neuropathy. Elevated intraocular pressure, especially when unresponsive to medical therapy, may also require a
canthotomy/cantholysis. Limited ocular motility with diplopia, presence of subconjuntival hemorrhage, or onset of proptosis
may suggest orbital hemorrhage, but this alone is not typically require urgent intervention.
Question 86 of 130
A 30-year-old woman is referred to you by her neurosurgeon after removal of a cerebellar hemangioma. Which of the
following are you most likely to find?

Capillary hemangioma of the retina

Cavernous angioma of the retina

Racemous angioma of the retina

Cavernous hemangioma of the choroid


Please select an answer
Feedback: Capillary hemangioma of the retina is associated with cerebellar and other central nervous system
hemangiomas as part of the Von Hippel-Lindau syndrome. Choroidal hemangiomas are seen in Sturge-Weber
syndrome. Racemous angiomas of the retina and cavernous hemangiomas of the retina most commonly occur as isolated
entities.

Question 87 of 130

A 62-year-old diabetic and hypertensive male wakes the day after cardiac bypass graft and notices that the vision in his left
eye is blurry. That evening his right eye becomes similarly affected. On examination, vision is 20/40 OD and 20/60 OS. No
afferent defect is detected. However, there are bilateral inferior altitudinal visual field defects, and both optic discs are
swollen superiorly with splinter hemmorhages. Which test or mechanism best characterizes this disorder?

The patient's erythrocyte sedimentation rate (ESR) of 87 on the second postoperative day is probably a result of
temporal arteritis.

The patient probably experienced blood loss and periods of hypotension during the operation.

The absence of relative afferent pupillary defect means that vision loss is likely due to a vitreous hemorrhage.

Visual loss is related to embolic occlusion in the posterior ciliary circulations.


Please select an answer
Feedback: Anterior ischemic optic neuropathy (AION), while typically idiopathic and unilateral, can occur following
hypotension, blood loss, or general surgical procedures (most commonly back and cardiac surgery). Under these
circumstances, the anterior ischemic optic neuropathy is bilateral about half the time. If the severity of AION is symmetric,
there may be no relative afferent pupillary defect. Emboli are not thought to be a cause of anterior ischemic optic neuropathy
in most patients. In the perioperative period, the erythrocyte sedimentation rate may be elevated, and in the absence of
other symptoms, should not, in isolation, be thought to be caused by temporal cell arteritis.

Question 88 of 130
Rebound nystagmus is a sign for a lesion of what structure?

Medial longitudinal fasiculus (MLF)

Parapontine reticular formation

Rostral interstitial nucleus of the MLF

Cerebellar hemisphere
Please select an answer
Feedback: Rebound nystagmus occurs with lesions involving the cerebellum. Lesions involving the medial longitudinal
fasciculus cause internuclear ophthalmoplegia. The rostral interstitial nucleus of the MLF is involved in coordinating vertical
gaze, and the parapontine reticular formation is involved in horizontal gaze, so lesions at these sites result in gaze palsies.

Question 89 of 130
Compared to patients who have optic neuritis, patients with neuroretinitis are at low risk for developing which of the following
conditions?

Optic disc swelling associated with exudates

Multiple sclerosis

Lyme disease

Permanent vision loss


Please select an answer
Feedback: Neuroretinitis, optic disc edema with macular exudate, often in the configuration of a macular star, is most
commonly seen from inflammatory optic neuropathies, including those from infection such as cat scratch disease or Lyme
disease. Often there are vitreous cells, and visual function typically improves spontaneously. Neuroretinitis, marked optic
disc edema, and marked optic disc hemorrhage or macular exudates are not typically seen in patients who have
demyelinating optic neuritis from multiple sclerosis.

Question 90 of 130
A 30-year-old female complains of a continuous quivering of the upper and lower lids around her right eye over the last 2
weeks. On examination, there is a continuous undulation in the right orbicularis oculi that trails off into adjacent facial
muscles. There is also an asymetrically prominent nasolabial fold on the right, and her palpebral fissure is narrower on the
right side than on the left. She has no facial weakness. Ophthalmoscopy reveals that the patient has bilateral temporal pallor
and nerve fiber thinning in both eyes. Her saccadic eye movements show mild adduction slowing with abductor overshoot.
What is the most likely diagnosis?

Multiple sclerosis

Mitochondrial myopathy

Guillain-Barre syndrome

Brainstem glioma
Please select an answer
Feedback:

The description of undulating movements of the eyelid are characteristic of eyelid myokymia, most commonly a benign and
self-limited condition when it occurs in isolation. Extension of the myokymia to involve other facial muscles indicates more
widespread involvement of the facial nerve, most commonly seen in ponitine lesions. The presence of optic disc pallor, if
caused by the same condition, is likely related to an optic neuropathy, and of the listed conditions, only mutiple sclerosis
would be associated with a combination of facial myokymia and optic neuropathy.
Question 91 of 130
Substernal chest pain in a patient with anterior ischemic optic neuropathy (AION) caused by giant cell arteritis could indicate
what condition?

Hepatomegaly

Coronary arteritis

Toxic steroid levels

Referred pain from carotid arteritis


Please select an answer
Feedback:

Temporal (giant cell) arteritis is a systemic vasculitis that involves medium and small arteries. Although the most common
ophthalmic manifestations involve the ophthalmic circulation, other circulations including the cerebral arteries and coronary
arteries can be involved. High steriod levels would most likely reduce arteritis. Carotid arteritis does not cause referred
pain. Hepatomegaly is unrelated to ophthalmic arteritis.

Question 92 of 130

Which of the following therapies is preferred when managing non-arteritic anterior ischemic optic neuropathy (NAION)?

Aspirin protects the contralateral eye.

Lowering intraocular pressure (IOP) for visual loss.

No effective treatment has been proven.

Optic nerve sheath decompression for visual loss.


Please select an answer
Feedback: No therapy for NAION has been proven to be effective. The optic nerve decompression trial, which randomized
patients with NAION to observation or surgery, was not only unhelpful, but potentially harmful. Lowering IOP has not been
found to help, and most patients have normal IOP in the affected and fellow eye. Aspirin has not been proven to be effective
acutely or to prevent involvement of the fellow eye. So far, agents used for neuroprotection have not been effective in
preventing NAION or shown to reduce visual loss.

Question 93 of 130

Following injection of intravenous edrophonium (tensilon), what finding would most strongly suggest the diagnosis of
myasthenia gravis?

Nystagmus resolution.

Saccadic movements return to normal velocity.

Ocular misalignment improves with alternate cover testing.

Eyelid myokymia.
Please select an answer
Feedback: Intravenous edrophonium is useful diagnostically for myasthenia gravis because it improves neuromuscular
transmission. The most predictive findings for a positive response to edrophonium are objective improvements in ptosis or
ocular misalignment. Saccadic speed is often normal, and nystagmus does not occur from myasthenia gravis.

Question 94 of 130
When managing a patient with thyroid orbitopathy who has developed vision loss, what procedure or intervention should be
included?

Serial computed tomography (CT) scans of the orbit every 3 months

Periodic measurement of visual fields and color vision

Topical steroid therapy and systemic diuresis

Strabisumus surgery soon after the development of double vision


Please select an answer
Feedback: Thyroid orbitopathy may result in optic neuropathy (estimated to occur in 5% of patients with thyroid eye
disease). Due to the potential for optic neuropathy, color vision and visual fields should be measured and followed over
time. Although CT scans of the orbit help in diagnosing thyroid eye disease and establishing compressive optic neuropathy
as a cause of visual loss (most commonly from crowding at the orbital apex secondary to enlargement of the extraocular
muscles), repeating them every 3 months serially is not necessary. Systemic corticosteroids may play a role in decreasing
the inflammatory component of thyroid orbitopathy, but topical corticosteroids are not helpful. Strabismus surgery, if
necessary, should be delayed until after orbital decompression and after the ocular misalignment is stable.

Question 95 of 130
When considering a diagnosis of idiopathic intracranial hypertension (IIH or pseudotumor cerebri), treatment with what
antibiotic may be relevant?

Ciprofloxicin

Gentamicin

Fluconazole

Tetracycline
Please select an answer
Feedback:

Elevated intracranial pressure can result from a number of causes including mass lesions, hydrocephalus, meningitis, and
thrombosis of the cerebral venous sinuses. Some medications have been associated with elevated intracranial pressure,
including tetracycline and its derivatives. Gentamicin, Ciprofloxin and fluconazole are not implicated as a cause of elevated
intracranial pressure.
Question 96 of 130
A central scotoma in one eye and a superior temporal defect in the other suggests a lesion in what location?

Junction of the optic nerve and chiasm

Both optic nerves

Occipital lobe

Optic tract
Please select an answer
Feedback:

Preferential involvement of the temporal visual field suggests chiasmal disease until proven otherwise. The combination of a
central visual field defect in one eye and a superotemporal defect in the other eye (termed a junctional scotoma) localizes
the problem to the anterior optic chiasm, the junction between the intracranial optic nerve and optic chiasm. Bilateral optic
nerve disease would be expected to cause bilateral central defects or nerve fiber bundle type defects. Lesions of the optic
tract and parietal lobe result in homonymous hemianopias.

Question 97 of 130

What is the structural basis for the pupillary light-near dissociation in Adie's tonic pupil?

More fibers innervate the ciliary body than the pupillary sphincter.

More fibers innervate the pupillary sphincter than the ciliary body.

The ciliary ganglion supplies the sympathetic innervation to the pupil and results in a smaller ipsilateral pupil.

Fibers innervating the pupil are aberrantly activated with contraction of the extraocular muscles.
Please select an answer
Feedback: Tonic (Adie's) pupils are thought to occur from loss of innervation of the pupillary sphincter from the ciliary
ganglion. Many more fibers are targeted for the ciliary body than for the pupillary sphincter muscle, and loss of innervation
from the ciliary ganglion results in light-near dissociation, with relative preservation of pupil constriction with near response
compared to light response. The ciliary ganglion provides parasympathetic imput to the pupillary sphincter. Aberrant
pupillary constriction with extraocular movement may be seen after third nerve palsy due to aberrant regeneration, not with
tonic pupils.

Question 98 of 130
A 65-year-old man presents for a routine examination. The pupils are each 4 millimeters in diameter, round, and poorly
reactive to direct and consensual light. A brisk near response is present with brisk redilation on refixation from near to far.
Which of the following testing devices or procedures is most likely to yield diagnostic information?

Reflex hammer

Lumbar puncture

Dilute pilocarpine

Optokinetic drum
Please select an answer
Feedback: For patients who have pupils that respond sluggishly to light, the pupillary response to near stimulus should be
assessed. Light-near dissociation most commonly occurs from causes of decreased vision such as bilateral retinal or optic
nerve disease. This finding can also be seen in dorsal midbrain (Parinaud) syndrome, characterized by deficits in upgaze,
light near dissociation, and convergence retraction. Pursuit and saccades can be appreciated using an optokinetic drum,
which can be rotated downward to produce upward saccades. Upward saccades in patients with dorsal midbrain syndrome
often induce convergence-retraction of the globes. Tonicity to near is suggestive of tonic (Adie) pupils, and otherwise is not
associated with reduced visual function or an upgaze deficit. Dilute pilocaprine is often associated with supersensitivity with
tonic pupils.

Question 99 of 130

A 42-year-old woman had endovascular coils placed in a left posterior communicating aneurysm 3 months ago. She now
has a nearly complete, pupil-involving third-nerve palsy. What would be the preferred management?

Patching

Surgical repair of ptosis

Referral for strabismus surgery

Trial of corticosteroids to reduce inflammation from coiling


Please select an answer
Feedback: Coiling of intracranial aneurysms has been increasingly used as a primary mode of treatment. This includes
treatment of aneurysms involving the posterior communicating artery, the most common to cause a pupil-involving third-
nerve palsy. Improvement of the cranial nerve palsy may occur as coils cause thrombosis and decrease pulsations within
the aneurysmal sac. Resolution of cranial nerve palsy may take months to occur, therefore early surgery for ptosis and
ocular misalignment should be avoided. Corticosteroids are not generally used because coiling is not typically associated
with inflammation.

Question 100 of 130


In a patient with papilledema, what do the small, bright, refractile lesions on the optic disc represent?

Pseudodrusen of chronic swelling

Early resolution of the swelling

Neovascularization

Neuroretinitis
Please select an answer
Feedback: Chronic papilledema may result in chronic blockage of axoplasmic flow, which results in visible refractile-
appearing areas superificially, within the optic disc. These areas are termed pseudodrusen because they are not true optic
disc drusen, characterized by calcium deposition. Pseudodrusen typically take months to develop and do not occur with
early optic disc resolution. Neither neovascularization of the optic disc, which causes abnormal blood vessel formation, nor
neuroretinitis, which implies the formation of macular exudate in combination with optic disc edema, are associated with
refractile lesions of the optic disc.
Question 101 of 130
Chronic treatment of giant cell arteritis with corticosteroids can affect mobility by causing what common systemic
complication?

Hypotension with syncope

Myopathy

Chronic arthritis

Hypoglycemia with weakness


Please select an answer
Feedback: Chronic corticosteroids may cause myopathy that can limit mobility. They may also cause osteoporosis,
hypertension, and diabetes mellitus.

Question 102 of 130


What effect on the course of demyelinating optic neuritis, do corticosteroids have?

Increase the number of recurrent episodes if given intravenously

Improve the long-term (15-year) visual outcome

Speed the onset of visual recovery

Decrease the long-term (15-year) risk of clinically definite multiple sclerosis


Please select an answer
Feedback: The Optic Neuritis Treatment Trial has shown that corticosteroids, when given intravenously followed by an oral
taper, speed the onset of visual recovery, but do not alter the final visual outcome or risk of a subsequent diagnosis of
multiple sclerosis. In the same study, oral corticosteroids were found to be associated with a higher risk of recurrent optic
neuritis in the same and fellow eye. An increase in the recurrence rate of optic neuritis was noted with oral, no intravenous,
corticosteroids. At the 15 year time point after optic neuritis there was no significant difference in visual outcome or in the
risk of clinically definite multiple sclerosis between the oral and intravenous groups.

Question 103 of 130


Which of the following conditions is most likely to be associated with retinal vein occlusion?

Retinal cavernous hemangioma

Carotid cavernous fistula

Midbrain arteriovenous malformation

Cerebellar hemangioblastoma
Please select an answer
Feedback: Carotid cavernous fistulas are associated with retinal vein occlusion, presumably mediated by venous
congestion. Midbrain arteriovenous malformations can be associated with arteriovenous malformations in the retina
(Wyburn-Mason syndrome), but not with retinal vein occlusions. Retinal cavernous hemangiomas may hemorrhage but not
typically associated with retinal vein occlusion. Cerebellar hemangioblastomas are associated with retinal angiomas
(hamartomas), which are usually associated with dilated feeder artery(s) and vein(s).
Question 104 of 130
Which of the following conditions would be most likely to cause a homonymous hemianopia in a 83-year-old man?

Stroke

Mass lesion

Infection

Demyelination
Please select an answer
Feedback: While all of the entities listed may cause a homonymous hemianopia, stroke is by far the most common cause in
the elderly. The new onset of a homymous hemianopia, even in the absence of other symptoms, should prompt an urgent
evaluation for stroke in these patients.

Question 105 of 130

How are the trochlear nerves (fourth cranial nerve) related to their origins (nuclei) and targets?

The fourth cranial nerve exits the brain stem ventrally and passes uncrossed to the superior oblique muscle

They decussate in the brainstem, exit ventrally and pass uncrossed to the superior oblique muscles.

The fourth cranial nerve exits the brain stem ventrally and crosses the midline ventral to the brainstem

They exit the brainstem dorsally, cross sides and pass to the superior oblique muscles.

The fourth cranial nerve decussates within the brainstem exits the brain stem dorsally and crosses to the contralateral
superior oblique muscle

They decussate within the brainstem, exit dorsally and pass uncrossed to the superior oblique muscles.

The fourth cranial nerve exits the brain stem dorsally and passes uncrossed to the superior oblique muscle

They exit the brainstem dorsally, cross sides and then pass to the superior oblique muscles.

Please select an answer


Feedback:

The fourth cranial nerve is unique in that it decussates in the brainstem, so that the fourth nerve nucleus on the right gives
rise to the left fourth nerve fascicle. It is the smallest cranial nerve and has the longest course, making it prone to damage
with head trauma.
Question 106 of 130
A 40-year-old man has developed a central retinal vein occlusion (CRVO) in the right eye. He does not have systemic
hypertension or diabetes mellitus. What additional testing would have the highest likelihood to reveal an underlying
systemic cause?

Serum electrolytes

Tests for hypercoagulable disorders.

Liver function tests

Genetic testing
Please select an answer
Feedback: Central retinal vein occlusion may be the initial manifestation of a hypercoagulable disorder, especially in
younger patients with no known history of vascular risk factors, such as hypertension or diabetes mellitus. The remaining
tests have little likelihood of detecting a related systemic condition.

Question 107 of 130

For a patient with a unilateral internuclear ophthalmoplegia, in which field of gaze is the ocular misalignment largest?

Gaze ipsilateral to the brainstem lesion within the medial longitudinal fasciculus

Gaze contralateral to the brainstem lesion within the paramedian pontine reticular formation

Gaze contralateral to the brainstem lesion within the medial longitudinal fasciculus (MLF)

Gaze ipsilateral to the brainstem lesion within the paramedian pontine reticular formation (PPRF)
Please select an answer
Feedback: Internuclear ophthalmoplegia occurs with lesions of the MLF, which runs from the PPRF in the pons to the
midbrain to innervate the medial rectus subnucleus. The MLF decussates in the pons so that innervation from the right
PPRF travels to the left medial rectus third nerve subnucleus. Lesions of the PPRF cause an ipsilateral gaze palsy and not
internuclear ophthalmoplegia.

Question 108 of 130


Which of the following findings is commonly found on MR or CT scans from patients with papilledema?

Flattening of the posterior portions of the globes

Absence of the posterior pituitary bright spot

Hyperdensity within the optic nerves consistent with calcium

Enhancement of the optic nerve sheaths


Please select an answer
Feedback:

Papilledema (optic disc edema secondary to elevated intracranial pressure) may be associated with findings on
neuroimaging studies including flattening of the posterior portion of the globes and flattening of the pituitary gland with an
empty sella. Optic nerve sheath enhancement is expected with inflammation of the optic nerves or optic nerve
tumors. Hyperdensity consistent with calcium on computed tomography is suggestive of optic disc drusen. Absence of the
posterior pituitary bright spot can be seen in midline developmental abnormalities such as septo-optic dysplasia.

Question 109 of 130


The double Maddox rod test is used to measure what abnormality or characteristic?

Torsional misalignment

Anomalous retinal correspondence

Over minus during refraction

Degree of esotropia
Please select an answer
Feedback: Double maddox rods are typically used to test for torsion. A virtual image, perpendicular to the plane of the
maddox rod, is seen at the source of light, held at distance, where the two rods should be parallel. In patients with abnormal
torsion, the rods are not parallel and the rods can be twisted to make them parallel. The resulting degree of torsion can then
be quantified.

Question 110 of 130


What measure of vision is frequently abnormal in patients with optic disc drusen?

Photostress recovery test

Visual acuity

Peripheral visual fields

Color vision
Please select an answer
Feedback: Optic disc drusen most commonly cause visual field defects without affecting central visual acuity or color
vision. Photostress recovery may be abnormal in maculopathy but is not typically affected in optic neuropathy caused by
optic disc drusen.

Question 111 of 130


An HIV-positive man has bilateral disc swelling, choroiditis and retinal phlebitis. What is the most likely diagnosis for his
ocular findings?

Vogt-Koyangi-Harada (VKH) syndrome

Lyme disease

Behcet's disease

Syphilis
Please select an answer
Feedback: The incidence of syphilis is higher in those with HIV. Syphilis can cause virtually any type of intraocular
inflammation including vitritis, optic disc edema, retinitis, choroiditis, and vasculitis. Lyme disease, VKH, and Bechet's
disease are rare and not significantly increased in HIV-positive individuals.

Question 112 of 130


What finding is likely to be found in a patient with acute optic neuritis due to multiple sclerosis?

Vitritis in the ipsilateral eye with optic neuritis

A paradoxically normal optic disc in the eye affected by optic neuritis

Optic disc edema in the contralateral eye

Meningeal enhancement over the temporal lobe on T1-weighted MRI


Please select an answer
Feedback: Acute optic neuritis associated with multiple sclerosis is more commonly retrobulbar and therefore if seen early
in the course is associated with a normal optic disc appearance. Demyelinating optic neuritis is typically unilateral and not
associated with uveitis, or vitritis. If meningeal enhancement is seen, other inflammatory conditions such as sarcoidosis and
infectious causes should be considered.

Question 113 of 130


A 17-year-old patient has acute onset of diplopia associated with pain, chemosis and proptosis in one eye. The CT scan of
the left eye shows uniform enlargement of the lateral rectus muscle, but no enlargement of any other extraocular muscles.
What is the most likely diagnosis?

Orbital cellulitis

Myositis

Thyroid eye disease

Extraocular muscle metastases


Please select an answer
Feedback: Isolated enlargement of a single rectus muscle would be typical of myositis. In the absence of other evidence of
an infection, orbital cellulitis causing mysositis would be unlikely. Metastases in a 17-year-old without a known primary
cancer would be unlikely. Thyroid eye disease tends to involve multiple muscles and typically spares the tendon while
involving the muscle belly.
Question 114 of 130
Multiple sclerosis is least likely to occur in a patient who develops optic neuritis and has which of the following?

Exudate in the pattern of a macular star

Multiple periventricular lesions on brain MRI

Pain with eye movement

Normal optic disc at onset of visual loss


Please select an answer
Feedback:

Demyelinating optic neuritis, the form associated with multiple sclerosis, commonly causes pain with eye movement.
Periventricular white matter lesions, typical for mutiple sclerosis, are common in optic neuritis and are the most important
risk factor for the development of subsequent neurologic dysfunction. Acutely the optic disc is normal in 2/3 of patients with
demyelinating optic neuritis, and marked hermorrhage and macular exudate are atypical.

Question 115 of 130


What clinical feature is most helpful in differentiating a chronic, pharmacologically constricted pupil from one affected by
tertiary syphilis?

Pupillary response to light

Appearance of pupillary sphincter

Pupillary response to near

Appearance of iris stroma


Please select an answer
Feedback:

Pupillary dysfunction from syphilis is characterized by light-near dissociation. Syphilitic pupils constrict better to a near
target than to direct light stimulation. Pharmacologic constriction with pilocarpine stimulates the iris constrictor muscle.
Therefore activity of the pupil is affected both at near and to light. The appearance of the iris stroma and sphincter may be
normal, or restricted by senechia, in both conditions.

Question 116 of 130


A lesion in what location typically results in a dilated pupil that constricts with 1/8% pilocarpine?

Iris sphincter

Cavernous sinus

Ciliary ganglion

Midbrain
Please select an answer
Feedback: Tonic pupils are thought to result from lesions affecting the ciliary ganglion. These pupils react better to light than
near, are tonic to a near stimulus, and often are supersensitive to pilocarpine, constricting to 1/8%. Lesions of the midbrain
and cavernous sinus would be expected to cause a 3rd-nerve palsy, potentially with pupil involvement. Iris sphincter
damage would cause a sluggish pupil at light and near and would not be expected to constrict with 1/8% pilocarpine.

Question 117 of 130


A patient with temporal arteritis, an elevated elevated sedimentation rate and and abnormal C-reactive protein, Often has
what other laboratory abnormality?

Thrombocytosis

Hypercalcemia

Erythrocytosis

Elevated angiotensin-converting enzyme


Please select an answer
Feedback: Giant cell (temporal) arteritis is a chronic inflammatory condition associated with abnormalities in laboratory
values. Elevation in the sedimentation rate and C-reactive protein are common. An elevated platelet count and anemia are
commonly present.

Question 118 of 130


A woman had idiopathic intracranial hypertension (pseudotumor cerebri). What would be the most common early finding?

Central scotomas on visual-field testing

Elevated protein on CSF analysis

Ventriculomegaly on MRI of the brain

Bilateral optic disc edema


Please select an answer
Feedback: The diagnostic criteria of idiopathic intracranial hypertension include normal cerebrospinal fluid consistency and
no evidence of structural abnormalities such as enlarged ventricles. It is a diagnosis of exclusion. Early on, visual function is
excellent in patients with papilledema, and a central scotoma would be unexpected. Elevated protein in the CSF would
suggest an alternative cause of elevated intracranail pressure such as inflammatory or neoplastic disorders.
Question 119 of 130
A 30-year-old woman has bilateral optic atrophy, bilateral deafness, cataracts, and premacular membranes. Assuming that
all of these findings are due to a single condition, for what condition should she be evaluated?

Pseudotumor cerebri

Neurofibromatosis type II

Multiple sclerosis

Joubert's syndrome
Please select an answer
Feedback: This constellation of findings may be found in Usher's syndrome, but one would expect to find other features of
retinitis pigmentosa. Bilateral posterior subcapsular cataracts and decreased hearing may also be found in type II
neurofibromatosis, which has also been associated with bilateral acoustic neuromas. Epiretinal membranes may also be
present in these individuals. Joubert's syndrome is associated with ocular motor apraxia and cerebellar abnormalities.
Pseudotumor cerebri causes optic disc edema because of papilledema. Optic atrophy may be found late in the disorder, but
the other features mentioned are not found in pseudotumor cerebri. Multiple sclerosis is associated with inflammatory optic
neuropathy, but does not commonly affect hearing.

Question 120 of 130


What is the most likely lesion site to cause a nuclear 3rd-nerve palsy?

Pons, ipsilateral to the palsy

Midbrain, ipsilateral to the palsy

Midbrain, contralateral to the palsy

Pons, contralateral to the palsy


Please select an answer
Feedback: The midbrain is the site of the 3rd-nerve nucleus, which is composed of several subnuclei. The nerve fascicle
arising from these subnuclei travels out of the midbrain to enter the ipsilateral cavernous sinus and orbit.
Question 121 of 130

A 56-year-old woman presents with unilateral painful proptosis and diplopia. What additional finding would prompt
consideration of treatment with systemic steroids?

Recent ipsilateral strabismus surgery

Lid and conjuntival hemorrhage following recent altercation

"S" Shaped lid margin profile and fullness of superior temporal orbit

Enlargement of the lateral rectus muscle on MRI of the orbit


Please select an answer
Feedback: Enlargement of the lateral rectus with pain and diplopia is suggestive of orbital inflammation (orbital
pseudotumor) for which systemic corticosteroids would be the primary intervention. Lid and conjunctival hemorrhage,
especially associated with a history of recent trauma would suggest an orbital hemorrhage, which would not typically be
treated with systemic steroids. Recent ipsilatersl strabismus or orbital surgery would suggest that an infectious etiology
should be considered. A fullness of the superior temporal orbit associated with an "S" shaped lid contour suggests a
lacrimal tumor. Painful lacrimal tumors would bring a the diagnosis of adenoid cystic carcinoma into consideration.

Question 122 of 130


During the evaluation of a patient with headache, what characteristic(s) would suggest increased intracranial pressure?

Resolution of head pain in the morning

Scintillating scotoma

Hemicranial head pain that throbs

Bifrontal head pain, intracranial noises


Please select an answer
Feedback: Elevated intracranial pressure is typically worse on awakening. Scintillating scotomas and hemicranial, throbbing
head pain are most commonly seen with migraine. Intracranial noises may occur because of turbulent flow within the
cerebral venous sinuses that may occur with elevated intracranial pressure or from a fistula.
Question 123 of 130
A patient with a left eye abduction defect has normal pupil size and reactions, normal forced duction testing, and normal
saccadic velocity of the left lateral rectus muscle. Which of the following causes is the most likely?

6th-nerve palsy

Thyroid eye disease

Myasthenia gravis

Accomodative spasm
Please select an answer
Feedback:

Normal forced duction testing implies the problem is not due to restriction, making thyroid eye disease less likely.
Accomodative spasm is associated with pupillary miosis and variable esotropia. A 6th-nerve palsy is typically associated
with decreased saccades of the involved lateral rectus muscle. Myasthenia gravis does not typically involve the pupil and
does not cause restriction. The saccadic velocity is most commonly normal clinically, but can be impaired as more severe
ophthalmoplegia develops.

Question 124 of 130


What extraocular muscle(s) is innervated by the ipsilateral oculomotor subnuclei?

Ipsilateral superior rectus muscle

Contralateral inferior rectus muscle

Both superior rectus muscles

Contralateral superior rectus muscle


Please select an answer
Feedback: The 3rd-nerve motor nucleus include the subnuclei and fascicles to the superior rectus, inferior rectus, medical
rectus, and inferior oblique. The superior rectus subnucleus innervates the contralateral superior rectus, and all the other
subnuclei are uncrossed, and so innervate ipsilateral muscles. There is a single, midline nucleus that innervates both levator
palpebrae superioris muscles.

Question 125 of 130


A 52-year-old diabetic man develops a partial 3rd-nerve palsy. His eyelid elevates when he looks downward. What is his
most likely diagnosis?

Elevated intracranial pressure

Compressive lesion

Midbrain stroke

Ischemic (vasculopathic) cranial mononeuropathy


Please select an answer
Feedback: When associated with a 3rd-nerve palsy, eyelid elevation on attempted downgaze suggests abberant
regeneration, which is most commonly associated with compressive lesions and trauma. It also can be seen in patients with
congenital 3rd-nerve palsy. Ischemic causes such as stroke and vasculopathic cranial nerve palsies do not cause abberant
regeneration. Elevated intracranial pressure rarely results in 3rd-nerve palsy, again without abberant regeneration.
Question 126 of 130
A 29-year-old pharmacist was parked when her car was struck from the side. Since the accident, her right pupil is small. In
the light, the pupils are 1 mm OD, 4.5 mm OS. In dim light, the pupils are 1 mm OD and 5.5 OS. The right pupil is unreactive
to light or near stimulus. The left pupil appears normal. The remainder of her ocular and extra-ocular examination is normal.
What's the most likely diagnosis for her pupillary anomaly?

Horner syndrome

Chronic uveitis with posterior synechiae

Tertiary syphilis

Pharmacologic anisocoria
Please select an answer
Feedback:

A pupil that fails to react to light and near is suggestive of an iris sphincter problem. The unusual feature in this case is that
the affected pupil is the smaller one. Anisocoria greater in the dark suggests that the sphincter muscle is tonically constricted
in the right eye, and this is usually due to pharmacologic constriction from pilocarpine or another muscarinic anticholinergic
drug. Posterior synechiae would not be found in a normal examination. Horner syndrome does not cause a poorly reactive
pupil and would be expected to cause other findings such as ptosis. Syphilic pupils are typically bilateral, irregularly shaped,
and associated with light-near dissociation.

Question 127 of 130


A 15-year-old girl awoke with a red, itchy left eye. That afternoon she developed left blurry vision and brow ache. Her pupils
measured 6 mm OD and 1 mm OS, in the light, and 7 mm OD and 1 mm OS, in the dark. The right pupil is briskly reactive;
the left does not react to light or near. Otherwise her examination is unremarkable. What would be the preferred
management of her condition?

Instill topical cocaine 10% in each eye

Emergent MRI brain and orbits with gadolinium

Reassurance and follow-up in 2 days

Serum RPR and FTA-ABS


Please select an answer
Feedback:

A pupil that fails to react to light and near is suggestive of an iris sphincter problem, even though that pupil is the smaller
one. Anisocoria greater in the dark is suggestive that the sphincter muscle is tonically constricted on the right, suggestive of
pharmacologic anisocoria with a muscarinic agent such as pilocarpine, which may also be causing accomodative spasm and
a brow ache, so that reassurance that the anisocoria will resolve is important and neuroimaging unnecessary. Horner
syndrome does not cause a poorly reactive pupil and would be expected to cause other findings such as ptosis, so that
pharmacologic testing would not be necessary. Syphilic pupils are typically bilateral, irregularly shaped, and associated with
light-near dissociation, so that serum testing would not be required.
Question 128 of 130
A 40-year old woman develops blurred vision at near in the left eye. She has a mid-dilated pupil that reacts only to near
stimulus. What is the most likely diagnosis?

Third cranial nerve palsy

Accommodative insufficiency

Adie's pupil

Accommodative spasm
Please select an answer
Feedback:

An isolated dilated pupil that reacts to near better than light is typically from a tonic (Adie) pupil. Loss of accomodation
frequently occurs with pupillary dilation. Accomodative insufficiency and accomodative spasm are not associated with
pupillary dilation. Third nerve palsy would be expected to have other features, including ptosis and an ocular misalignment.

Question 129 of 130


A 40-year-old man is experiencing a headache associated with visual scintillations that is occurring with increasing
frequency several times per week, what would the appropriate step be?

Offer an oral beta-blocker as therapy.

Obtain an MRI scan.

Review the patient's history for environmental precipitants.

Check the patient's sedimentation rate.


Please select an answer
Feedback: A change in the frequency or pattern of headache is one finding that suggests the need for neuroimaging, even
in patients with chronic headaches, to exclude an expanding compressive lesion. In the absence of other findings suggestive
of inflammation, an erythrocyte sedimentation rate is not likely to be helpful. Environmental precipitants and treatment with
oral beta-blockers may be helpful in patients with migraine, however in this patient, a compressive lesion should be excluded
first.
Question 130 of 130
Systemic corticosteroid administration is associated with what expected side effect?

Hypotony maculopathy

Vitritis

Systemic hypotension

Osteoporosis
Please select an answer
Feedback: Osteoporosis, systemic hypertension, cataracts, increased susceptibility to infection, and ocular hypertension
are all potential side effects of systemic corticosteroid therapy. Inflammation, including vitritis, would not be an expected side
effect.

Collected from www.aao.org (self assessment)


By Dr. AlBaraa AlQassimi

You might also like